Preview (15 of 99 pages)

NCLEX-RN
Part 1
1. A depressed client is seen at the mental health center for follow-up after an attempted suicide 1 week
ago. She has taken phenelzine sulfate (Nardil), a monoamine oxidase (MAO) inhibitor, for 7 straight days.
She states that she is not feeling any better. The nurse explains that the drug must accumulate to an
effective level before symptoms are totally relieved. Symptom relief is expected to occur within:
A. 10 days
B. 2-4 weeks
C. 2 months
D. 3 months
Answer: B
Explanation:
(A) This answer is incorrect. It can take up to 1 month for therapeutic effect of the medication.
(B) This answer is correct. Because MAO inhibitors are slow to act, it takes 2-4 weeks before
improvement of symptoms is noted.
(C) This answer is incorrect. It can take up to 1 month for therapeutic effect of the medication.
(D) This answer is incorrect. Therapeutic effects of the medication are noted within 1 month of drug
therapy.
2. Cystic fibrosis is transmitted as an autosomal recessive trait. This means that:
A. Mothers carry the gene and pass it to their sons
B. Fathers carry the gene and pass it to their daughters
C. Both parents must have the disease for a child to have the disease
D. Both parents must be carriers for a child to have the disease
Answer: D
Explanation:
(A) Cystic fibrosis is not an X-linked or sex-linked disease.
(B) The only characteristic on the Y chromosome is the trait for hairy ears.
(C) Both parents do not need to have the disease but must be carriers.
(D) If a trait is recessive, two genes (one from each parent) are necessary to produce an affected child.

3. A 24-year-old client presents to the emergency department protesting "I am God." The nurse identifies
this as a:
A. Delusion
B. Illusion
C. Hallucination
D. Conversion
Answer: A
Explanation:
(A) Delusion is a false belief.
(B) Illusion is the misrepresentation of a real, external sensory experience.
(C) Hallucination is a false sensory perception involving any of the senses.
(D) Conversion is the expression of intrapsychic conflict through sensory or motor manifestations.
4. In acute episodes of mania, lithium is effective in 1-2 weeks, but it may take up to 4 weeks, or even a
few months, to treat symptoms fully. Sometimes an antipsychotic agent is prescribed during the first few
days or weeks of an acute episode to manage severe behavioral excitement and acute psychotic symptoms.
In addition to the lithium, which one of the following medications might the physician prescribe?
A. Diazepam (Valium)
B. Haloperidol (Haldol)
C. Sertraline (Zoloft)
D. Alprazolam (Xanax)
Answer: B
Explanation:
(A) Diazepam is an antianxiety medication and is not designed to reduce psychotic symptoms.
(B) Haloperidol is an antipsychotic medication and may be used until the lithium takes effect.
(C) Sertraline is an antidepressant and is used primarily to reduce symptoms of depression.
(D) Alprazolam is an antianxiety medication and is not designed to reduce psychotic symptoms.
5. A violent client remains in restraints for several hours. Which of the following interventions is most
appropriate while he is in restraints?
A. Give fluids if the client requests them.
B. Assess skin integrity and circulation of extremities before applying restraints and as they are removed.
C. Measure vital signs at least every 4 hours.

D. Release restraints every 2 hours for client to exercise.
Answer: D
Explanation:
(A) Fluids (nourishment) should be offered at regular intervals whether the client requests (or refuses)
them or not.
(B) Skin integrity and circulation of the extremities should be checked regularly while the client is
restrained, not only before restraints are applied and after they are removed.
(C) Vital signs should be checked at least every 2 hours. If the client remains agitated in restraints, vital
signs should be monitored even more closely, perhaps every 1-2 hours.
(D) Restraints should be released every 2 hours for exercise, one extremity at a time, to maintain muscle
tone, skin and joint integrity, and circulation.
6. The paediatrician has diagnosed tinea capitis in an 8- year-old girl and has placed her on oral
griseofulvin. The nurse should emphasize which of these instructions to the mother and/or child?
A. Administer oral griseofulvin on an empty stomach for best results.
B. Discontinue drug therapy if food tastes funny.
C. May discontinue medication when the child experiences symptomatic relief.
D. Observe for headaches, dizziness, and anorexia.
Answer: D
Explanation:
(A) Giving the drug with or after meals may allay gastrointestinal discomfort. Giving the drug with a fatty
meal (ice cream or milk) increases absorption rate.
(B) Griseofulvin may alter taste sensations and thereby decrease the appetite. Monitoring of food intake is
important, and inadequate nutrient intake should be reported to the physician.
(C) The child may experience symptomatic relief after 4896 hours of therapy. It is important to stress
continuing the drug therapy to prevent relapse (usually about 6 weeks).
(D) The incidence of side effects is low; however, headaches are common. Nausea, vomiting, diarrhoea,
and anorexia may occur. Dizziness, although uncommon, should be reported to the physician.
7. A client with cirrhosis of the liver becomes comatose and is started on neomycin 300 mg q6h via
nasogastric tube. The rationale for this therapy is to:
A. Prevent systemic infection
B. Promote diuresis

C. Decrease ammonia formation
D. Acidify the small bowel
Answer: C
Explanation:
(A) Neomycin is an antibiotic, but this is not the Rationale for administering it to a client in hepatic coma.
(B) Diuretics and salt-free albumin are used to promote diuresis in clients with cirrhosis of the liver.
(C) Neomycin destroys the bacteria in the intestines. It is the bacteria in the bowel that break down protein
into ammonia.
(D) Lactulose is administered to create an acid environment in the bowel. Ammonia leaves the blood and
migrates to this acidic environment where it is trapped and excreted.
8. A 5-year-old has just had a tonsillectomy and adenoidectomy. Which of these nursing measures should
be included in the postoperative care?
A. Encourage the child to cough up blood if present.
B. Give warm clear liquids when fully alert.
C. Have child gargle and do toothbrushing to remove old blood.
D. Observe for evidence of bleeding.
Answer: D
Explanation:
(A) The nurse should discourage the child from coughing, clearing the throat, or putting objects in his
mouth. These may induce bleeding.
(B) Cool, clear liquids may be given when child is fully alert. Warm liquids may dislodge a blood clot. The
nurse should avoid red- or brown-colored liquids to distinguish fresh or old blood from ingested liquid
should the child vomit.
(C) Gargles and vigorous toothbrushing could initiate bleeding.
(D) Postoperative haemorrhage, though unusual, may occur. The nurse should observe for bleeding by
looking directly into the throat and for vomiting of bright red blood, continuous swallowing, and changes
in vital signs.
9. An 80-year-old male client with a history of arteriosclerosis is experiencing severe pain in his left leg
that started approximately 20 minutes ago. When performing the admission assessment, the nurse would
expect to observe which of the following:
A. Both lower extremities warm to touch with 2_pedal pulses

B. Both lower extremities cyanotic when placed in a dependent position
C. Decreased or absent pedal pulse in the left leg
D. The left leg warmer to touch than the right leg
Answer: C
Explanation:
(A) This statement describes a normal assessment finding of the lower extremities.
(B) This assessment finding reflects problems caused by venous insufficiency.
(C) Decreased or absent pedal pulses reflect a problem caused by arterial insufficiency.
(D) The leg that is experiencing arterial insufficiency would be cool to touch due to the decreased
circulation.
10. A 16-month-old infant is being prepared for tetralogy of Fallot repair. In the nursing assessment, which
lab value should elicit further assessment and requires notification of physician?
A. pH 7.39
B. White blood cell (WBC) count 10,000 WBCs/mm3
C. Haematocrit 60%
D. Bleeding time of 4 minutes
Answer: C
Explanation:
(A) Normal pH of arterial blood gases for an infant is 7.35-7.45.
(B) Normal white blood cell count in an infant is 6,000-17,500 WBCs/mm3.
(C) Normal haematocrit in infant is 28%-42%. A 60% haematocrit may indicate polycythemia, a common
complication of cyanotic heart disease.
(D) Normal bleeding time is 2-7 minutes.
11. A male client is experiencing extreme distress. He begins to pace up and down the corridor.
What nursing intervention is appropriate when communicating with the pacing client?
A. Ask him to sit down. Speak slowly and use short, simple sentences.
B. Help him to recognize his anxiety.
C. Walk with him as he paces.
D. Increase the level of his supervision.
Answer: C
Explanation:

(A) The nurse should not ask him to sit down. Pacing is the activity he has chosen to deal with his anxiety.
The nurse dealing with this client should speak slowly and with short, simple sentences.
(B) The client may already recognize the anxiety and is attempting to deal with it.
(C) Walk with the client as he paces. This gives support while he uses anxiety-generated energy.
(D) Increasing the level of supervision may be appropriate after he stops pacing. It would minimize selfinjury and/or loss of control.
12. Prior to an amniocentesis, a fetal ultrasound is done in order to:
A. Evaluate fetal lung maturity
B. Evaluate the amount of amniotic fluid
C. Locate the position of the placenta and fetus
D. Ensure that the fetus is mature enough to perform the amniocentesis
Answer: C
Explanation:
(A) Amniocentesis can be performed to assess for lung maturity. Fetal ultrasound can be used for
gestational dating, although it does not separately determine lung maturity.
(B) Ultrasound can evaluate amniotic fluid volume, which may be used to determine congenital anomalies.
(C) Amniocentesis involves removal of amniotic fluid for evaluation. The needle, inserted through the
abdomen, is guided by ultrasound to avoid needle injuries, and the test evaluates the position of the
placenta and the fetus.
(D) Amniocentesis can be performed as early as the 15th-17th week of pregnancy.
13. A 25-year-old client believes she may be pregnant with her first child. She schedules an obstetric
examination with the nurse practitioner to determine the status of her possible pregnancy. Her last
menstrual period began May 20, and her estimated date of confinement using Nagele's rule is:
A. March 27
B. February 1
C. February 27
D. January 3
Answer: C
Explanation:
(A)March 27 is a miscalculation.
(B) February 1 is a miscalculation.

(C) February 27 is the correct answer. To calculate the estimated date of confinement using Nagele's rule,
subtract 3 months from the date that the last menstrual cycle began and then add 7 days to the result.
(D) January 3 is a miscalculation.
14. A client is now pregnant for the second time. Her first child weighed 4536 g at delivery. The client's
glucose tolerance test shows elevated blood sugar levels. Because she only shows signs of diabetes when
she is pregnant, she is classified as having:
A. Insulin-dependent diabetes
B. Type II diabetes mellitus
C. Type I diabetes mellitus
D. Gestational diabetes mellitus
Answer: D
Explanation:
(A) Insulin-dependent diabetes mellitus, also known as type I diabetes, usually appears before the age of
30 years with an abrupt onset of symptoms requiring insulin for management. It is not related to onset
during pregnancy.
(B) Non-insulin-dependent diabetes (type II diabetes) usually appears in older adults. It has a slow onset
and progression of symptoms.
(C) This type of diabetes is the same as insulin-dependent diabetes.
(D) Gestational diabetes mellitus has its onset of symptoms during pregnancy and usually disappears after
delivery. These symptoms are usually mild and not life threatening, although they are associated with
increased fetal morbidity and other fetal complications.
15. A 44-year-old female client is receiving external radiation to her scapula for metastasis of breast
cancer.
Teaching related to skin care for the client would include which of the following?
A. Teach her to completely clean the skin to remove all ointments and markings after each treatment
B. Teach her to cover broken skin in the treated area with a medicated ointment.
C. Encourage her to wear a tight-fitting vest to support her scapula.
D. Encourage her to avoid direct sunlight on the area being treated.
Answer: D
Explanation:

(A) The skin in a treatment area should be rinsed with water and patted dry. Markings should be left intact,
and the skin should not be scrubbed.
(B) Clients should avoid putting any creams or lotions on the treated area. This could interfere with
treatment.
(C) Radiation therapy clients should wear loose fitting clothes and avoid tight, irritating fabrics.
(D) The area of skin being treated is sensitive to sunlight, and the client should take care to prevent sun
damage by avoiding direct sunlight and covering the area when she is in the sun.
16. The nurse is assisting a 4th-day postoperative cholecystectomy client in planning her meals for
tomorrow's menu. Which vitamin is the most essential in promoting tissue healing?
A. Vitamin C
B. Vitamin B1
C. Vitamin D
D. Vitamin A
Answer: A
Explanation:
(A) Vitamin C (ascorbic acid) is essential in promoting wound healing and collagen formation.
(B) Vitamin B1 (thiamine) maintains normal gastrointestinal (GI) functioning, oxidizes carbohydrates, and
is essential for normal functioning of nervous tissue.
(C) Vitamin D regulates absorption of calcium and phosphorus from the GI tract and helps prevent rickets.
(D) Vitamin A is necessary for the formation and maintenance of skin and mucous membranes. It is also
essential for normal growth and development of bones and teeth.
17. A 10-year-old client with a pin in the right femur is immobilized in traction. He is exhibiting behavioral
changes including restlessness, difficulty with problem solving, inability to concentrate on activities, and
monotony. Which of the following nursing implementations would be most effective in helping him cope
with immobility?
A. Providing him with books, challenging puzzles, and games as diversionary activities
B. Allowing him to do as much for himself as he is able, including learning to do pin-site care under
supervision
C. Having a volunteer come in to sit with the client and to read him stories
D. Stimulating rest and relaxation by gentle rubbing with lotion and changing the client's position
frequently

Answer: B
Explanation:
(A) These activities could be frustrating for the client if he is having difficulty with problem solving and
concentration.
(B) Selfcare is usually well received by the child, and it is one of the most useful interventions to help the
child cope with immobility.
(C) This may be helpful to the client if he has no visitors, but it does little to help him develop coping
skills.
(D) This will help to prevent skin irritation or breakdown related to immobility but will not help to prevent
behavioral changes related to immobility.
18. In client teaching, the nurse should emphasize that fetal damage occurs more frequently with ingestion
of drugs during:
A. First trimester
B. Second trimester
C. Third trimester
D. Every trimester
Answer: A
Explanation:
(A) Organogenesis occurs in the first trimester. Fetus is most susceptible to malformation during this
period.
(B) Organogenesis has occurred by the second trimester.
(C) Fetal development is complete by this time.
(D) The dangerous period for fetal damage is the first trimester, not the entire pregnancy.
19. On admission, the client has signs and symptoms of pulmonary edema. The nurse places the client in
the most appropriate position for a client in pulmonary edema, which is:
A. High Fowler
B. Lying on the left side
C. Sitting in a chair
D. Supine with feet elevated
Answer: A
Explanation:

(A) High Fowler position decreases venous return to the heart and permits greater lung expansion so that
oxygenation is maximized.
(B) Lying on the left side may improve perfusion to the left lung but does not promote lung expansion.
(C) Sitting in a chair will decrease venous return and promote maximal lung expansion. However, clients
with pulmonary edema can deteriorate quickly and require intubation and mechanical ventilation. If a
client is sitting in achair when this deterioration happens, it will be difficult to intervene quickly.
(D) The supine with feet elevated position increases venous return and will worsen pulmonary edema.
20. A client has returned to the unit from the recovery room after having a thyroidectomy. The nurse knows
that a major complication after a thyroidectomy is:
A. Respiratory obstruction
B. Hypercalcemia
C. Fistula formation
D. Myxedema
Answer: A
Explanation:
(A) Respiratory obstruction due to edema of the glottis, bilateral laryngeal nerve damage, or tracheal
compression from hemorrhage is a major complication after a thyroidectomy.
(B) Hypocalcemia accompanied by tetany from accidental removal of one or more parathyroid glands is a
major complication, not hypercalcemia.
(C) Fistula formation is not a major complication associated with a thyroidectomy. It is a major
complication with a laryngectomy.
(D) Myxedema is hypothyroidism that occurs in adults and is not a complication of a thyroidectomy. A
thyroidectomy client tends to develop thyroid storm, which is excess production of thyroid hormone.
21. The nurse should facilitate bonding during the postpartum period. What should the nurse expect to
observe in the taking-hold phase?
A. Mother is concerned about her recovery.
B. Mother calls infant by name.
C. Mother lightly touches infant.
D. Mother is concerned about her weight gain.
Answer: B
Explanation:

(A) This observation can be made during the taking-in phase when the mother's needs are more important.
(B) This observation can be made during the taking-hold phase when the mother is actively involved with
herself and the infant.
(C, D) This observation can be made during the taking-in phase.
22. A female client has just died. Her family is requesting that all nursing staff leave the room. The family's
religious leader has arrived and is ready to conduct a ceremony for the deceased in the room, requesting
that only family members be present. The nurse assigned to the client should perform the appropriate
nursing action, which might include:
A. Inform the family that it is the hospital's policy not to conduct religious ceremonies in client rooms
B. Refuse to leave the room because the client's body is entrusted in the nurse's care until it can be brought
to the morgue.
C. Tell the family that they may conduct their ceremony in the client's room; however, the nurse must
attend.
D. Respect the client's family's wishes.
Answer: D
Explanation:
(A) It is rare that a hospital has a specific policy addressing this particular issue. If the statement is true,
the nurse should show evidence of the policy to the family and suggest alternatives, such as the hospital
chapel.
(B) Refusal to leave the room demonstrates a lack of understanding related to the family's need to grieve in
their own manner.
(C) The nurse should leave the room and allow the family privacy in their grief.
(D) The family's wish to conduct a religious ceremony in the client's room is part of the grief process. The
request is based on specific cultural and religious differences dictating social customs.
23. A woman diagnosed with multiple sclerosis is disturbed with diplopia. The nurse will teach her to:
A. Limit activities which require focusing (close vision)
B. Take more frequent naps
C. Use artificial tears
D. Wear a patch over one eye
Answer: D
Explanation:

(A) Limiting activities requiring close vision will not alleviate the discomfort of double vision.
(B) Frequent naps may be comforting, but they will not prevent double vision.
(C) Artificial tears are necessary in the absence of a corneal reflex, but they have no effect on diplopia.
(D) An eye patch over either eye will eliminate the effects of double vision during the time the eye patch is
worn. An eye patch is safe for a person with an intact corneal reflex.
24. One of the most reliable assessment tools for adequacy of fluid resuscitation in burned children is:
A. Blood pressure
B. Level of consciousness
C. Skin turgor
D. Fluid intake
Answer: B
Explanation:
(A) Blood pressure can remain normotensive in a state of hypovolemia.
(B) Capillary refill, alterations in sensorium, and urine output are the most reliable indicators for assessing
hydration.
(C) Skin turgor is not a reliable indicator for assessing hydration in a burn client.
(D) Fluid intake does not indicate adequacy of fluid resuscitation in a burn client.
25. Which one of the following is considered a reliable indicator for assessing the adequacy of fluid
resuscitation in a 3-year-old child who suffered partial- and full thickness burns to 25% of her body?
A. Urine output
B. Edema
C. Hypertension
D. Bulging fontanelle
Answer: A
Explanation:
(A) Urinary output is a reliable indicator of renal perfusion, which in turn indicates that fluid resuscitation
is adequate. IV fluids are adjusted based on the urinary output of the child during fluid resuscitation.
(B) Edema is an indication of increased capillary permeability following a burn injury.
(C) Hypertension is an indicator of fluid volume excess.
(D) Fontanelles close by 18 months of age.

26. A client with a diagnosis of C-4 injury has been stabilized and is ready for discharge. Because this
client is at risk for autonomic dysreflexia, he and his family should be instructed to assess for and report:
A. Dizziness and tachypnea
B. Circumoral pallor and light headed ness
C. Headache and facial flushing
D. Pallor and itching of the face and neck
Answer: C
Explanation:
(A) Tachypnea is not a symptom.
(B) Circumoral pallor is not a symptom.
(C) Autonomic dysreflexia is an uninhibited and exaggerated reflex of the autonomic nervous system to
stimulation, which results in vasoconstriction and elevated blood pressure.
(D) Pallor and itching are not symptoms.
27. A client states to his nurse that "I was told by the doctor not to take one of my drugs because it seems
to have caused decreasing blood cells." Based on this information, which drug might the nurse expect to be
discontinued?
A. Prednisone
B. Timolol maleate (Blocadren)
C. Garamycin (Gentamicin)
D. Phenytoin (Dilantin)
Answer: D
Explanation:
(A) Prednisone is not linked with haematological side effects.
(B) Timolol, a -adrenergic blocker is metabolized by the liver. It has not been linked to blood dyscrasia.
(C) Gentamicin is ototoxic and nephrotoxic.
(D) Phenytoin usage has been linked to blood dyscrasias such as aplastic anemia. The drug most
commonly linked to aplastic anemia is chloramphenicol (Chlormycetin).
28. A client has been taking lithium 300 mg po bid for the past two weeks. This morning her lithium level
was 1 mEq/L. The nurse should:
A. Notify the physician immediately
B. Hold the morning lithium dose and continue to observe the client

C. Administer the morning lithium dose as scheduled
D. Obtain an order for benztropine (Cogentin)
Answer: C
Explanation:
(A) There is no need to phone the physician because the lithium level is within therapeutic range and because
there are no indications of toxicity present.
(B) There is no reason to withhold the lithium because the blood level is within therapeutic range. Also, it is
necessary to give the medication as scheduled to maintain adequate blood levels.
(C) The lab results indicate that the client's lithium level is within therapeutic range (0.2-1.4 m Eq/L), so the
medication should be given as ordered.
(D) Benztropine is an ant parkinsonism drug frequently given to counter act extrapyramidal symptoms
associated with the administration of antipsychotic drugs (not lithium).
29. The primary focus of nursing interventions for the child experiencing sickle cell crisis is aimed toward:
A. Maintaining an adequate level of hydration
B. Providing pain relief
C. Preventing infection
D. O2 therapy
Answer: A
Explanation:
(A) Maintaining the hydration level is the focus for nursing intervention because dehydration enhances the
sickling process. Both oral and parenteral fluids are used.
(B) The pain is a result of the sickling process. Analgesics or narcotics will be used for symptom relief, but
the underlying cause of the pain will be resolved with hydration.
(C) Serious bacterial infections may result owing to splenic dysfunction. This is true at all times, not just
during the acute period of a crisis.
(D) O2 therapy is used for symptomatic relief of the hypoxia resulting from the sickling process. Hydration
is the primary intervention to alleviate the dehydration that enhances the sickling process.
30. Three weeks following discharge, a male client is readmitted to the psychiatric unit for depression. His
wife stated that he had threatened to kill himself with a handgun. As the nurse admits him to the unit, he
says, "I wish I were dead because I am worthless to everyone; I guess I am just no good." Which response
by the nurse is most appropriate at this time?

A. "I don't think you are worthless. I'm glad to see you, and we will help you."
B. "Don't you think this is a sign of your illness?"
C. "I know with your wife and new baby that you do have a lot to live for."
D. "You've been feeling sad and alone for some time now?"
Answer: D
Explanation:
(A) This response does not acknowledge the client's feelings.
(B) This is a closed question and does not encourage communication.
(C) This response negates the client's feelings and does not require a response from the client.
(D) This acknowledges the client's implied thoughts and feelings and encourages a response.
31. A 52-year-old client is scheduled for a small-bowel resection in the morning. In conjunction with other
preoperative preparation, the nurse is teaching her diaphragmatic breathing exercises. She will teach the
client to:
A. Inhale slowly and deeply through the nose until the lungs are fully expanded, hold the breath a couple
of seconds, and then exhale slowly through the mouth. Repeat 2-3 more times to complete the series every
1-2 hours while awake
B. Purse the lips and take quick, short breaths approximately 18-20 times/min
C. Take a large gulp of air into the mouth, hold it for 10-15 seconds, and then expel it through the nose.
Repeat 4-5 times to complete the series
D. Inhale as deeply as possible and then immediately exhale as deeply as possible at a rate of
approximately 20-24 times/min
Answer: A
Explanation:
(A) This is the correct method of teaching diaphragmatic breathing, which allows full lung expansion to
increase oxygenation, prevent atelectasis, and move secretions up and out of the lungs to decrease risk of
pneumonia.
(B) Quick, short breaths do not allow for full lung expansion and movement of secretions up and out of the
lungs. Quick, short breaths may lead to O2 depletion, hyperventilation, and hypoxia.
(C) Expelling breaths through the nose does not allow for full lung expansion and the use of diaphragmatic
muscles to assist in moving secretions up and out of the lungs.
(D) Inhaling and exhaling at a rate of 20-24 times/min does not allow time for full lung expansion to
increase oxygenation. This would most likely lead to O2 depletion and hypoxia.

32. The nurse enters the playroom and finds an 8-year-old child having a grand mal seizure. Which one of
the following actions should the nurse take?
A. Place a tongue blade in the child's mouth.
B. Restrain the child so he will not injure himself.
C. Go to the nurses station and call the physician.
D. Move furniture out of the way and place a blanket under his head.
Answer: D
Explanation:
(A) The nurse should not put anything in the child's mouth during a seizure; this action could obstruct the
airway.
(B) Restraining the child's movements could cause constrictive injury.
(C) Staying with the child during a seizure provides protection and allows the nurse to observe the seizure
activity.
(D) The nurse should provide safety for the child by moving objects and protecting the head.
33. An 11-year-old boy has received a partial-thickness burn to both legs. He presents to the emergency
room approximately 15 minutes after the accident in excruciating pain with charred clothing to both legs.
What is the first nursing action?
A. Apply ice packs to both legs.
B. Begin debridement by removing all charred clothing from wound.
C. Apply Silvadene cream (silver sulfadiazine).
D. Immerse both legs in cool water.
Answer: D
Explanation:
(A) Ice creates a dramatic temperature change in the tissue, which can cause further thermal injury.
(B) Charred clothing should not be removed from wound first. This creates further tissue damage.
Debridement is not the first nursing action.
(C) Applying silver sulfadiazine cream first insulates heat in injured tissue and increases potential for
infection.
(D) Emergency care of a thermal burn is immersing both legs in cool water. Cool water permits gradual
temperature change and prevents further thermal damage.

34. The nurse notes scattered crackles in both lungs and 1+ pitting edema when assessing a cardiac client.
The physician is notified and orders furosemide (Lasix) 80 mg IV push stat. Which of the following
diagnostic studies is monitored to assess for a major complication of this therapy?
A. Serum electrolytes
B. Arterial blood gases
C. Complete blood count
D. 12-Lead ECG
Answer: A
Explanation:
(A) Furosemide, a potassium-depleting diuretic, inhibits the reabsorption of sodium and chloride from the
loop of Henle and the distal renal tubules. Serum electrolytes are monitored for hypokalemia.
(B) Severe acid-base imbalances influence the movement of potassium into and out of the cells, but arterial
blood gases to not measure the serum potassium level.
(C) Furosemide is a potassium-depleting diuretic. A complete blood count does not reflect potassium
levels.
(D) Abnormalities in potassium (both hyperkalemia and hypokalemia) are reflected in ECG changes, but
these changes do not occur until the abnormality is severe.
35. A psychiatric client has been stabilized and is to be discharged. The nurse will recognize client insight
and behavioral change by which of the following client statements?
A. "When I get home, I will need to take my medicines and call my therapist if I have any side effects or
begin to hear voices."
B. "If I have any side effects from my medicines, I will take an extra dose of Cogentin."
C. "When I get home, I should be able to taper myself off the Haldol because the voices are gone now."
D. "As soon as I leave here, I'm throwing away my medicines. I never thought I needed them anyway."
Answer: A
Explanation:
(A) The client verbalizes that he is responsible for compliance and keeping the treatment team member
informed of progress. This behavior puts him at the lowest risk for relapse.
(B) Noncompliance is a major cause of relapse. This statement reflects lack of responsibility for his own
health maintenance.
(C) This statement reflects lack of insight into the importance of compliance.
(D) This statement reflects no insight into his illness or his responsibility in health maintenance.

36. A mother brings a 6-month-old infant and a 4-year-old child to the nursing clinic for routine
examination and screening. Which of these plans by the nurse would be most successful?
A. Examine the 4 year old first.
B. Provide time for play and becoming acquainted.
C. Have the mother leave the room with one child, and examine the other child privately.
D. Examine painful areas first to get them "over with."
Answer: B
Explanation:
(A) The 6 month old should be examined first. If several children will be examined, begin with the most
cooperative and less anxious child to provide modeling.
(B) Providing time for play and getting acquainted minimizes stress and anxiety associated with
assessment of body parts.
(C) Children generally cooperate best when their mother remains with them.
(D) Painful areas are best examined last and will permit maximum accuracy of assessment.
37. Diagnostic assessment findings for an infant with possible coarctation of the aorta would include:
A. A third heart sound
B. A diastolic murmur
C. Pulse pressure difference between the upper extremities
D. Diminished or absent femoral pulses
Answer: D
Explanation:
(A) S1 and S2 in an infant with coarctation of the aorta are usually normal. S3 and S4 do not exist with this
diagnosis.
(B) Either no murmur will be heard or a systolic murmur from an associated cardiac defect will be heard
along the left upper sternal border. A diastolic murmur is not associated with coarctation of the aorta.
(C) Pulse pressure differences of>20 mm Hg exist between the upper extremities and the lower
extremities. It is important to evaluate the upper and lower extremities with the appropriate- sized cuffs.
(D) Femoral and pedal pulses will be diminished or absent in infants with coarctation of the aorta.
38. During a client's first postpartum day, the nurse assessed that the fundus was located laterally to the
umbilicus. This may be due to:

A. Endometritis
B. Fibroid tumor on the uterus
C. Displacement due to bowel distention
D. Urine retention or a distended bladder
Answer: D
Explanation:
(A, B) Endometritis, urine retention, or bladder distention provide good distractors because they may delay
involution but do not usually cause the uterus to be lateral.
(C) Bowel distention and constipation are common in the postpartum period but do not displace the uterus
laterally.
(D) Urine retention or bladder distention commonly displaces the uterus to the right and may delay
involution.
39. An 80-year-old widow is living with her son and daughter- in-law. The home health nurse has been
making weekly visits to draw blood for a prothrombin time test. The client is taking 5 mg of coumadin per
day. She appears more debilitated, and bruises are noted on her face. Elder abuse is suspected. Which of
the following are signs of persons who are at risk for abusing an elderly person?
A. A family member who is having marital problems and is regularly abusing alcohol
B. A person with adequate communication and coping skills who is employed by the family
C. A friend of the family who wants to help but is minimally competent
D. A lifelong friend of the client who is often confused
Answer: A
Explanation:
(A) This answer is correct. Two risk factors are identified in this answer.
(B) This answer is incorrect. Persons at risk tend to lack communication skills and effective coping
patterns.
(C) This answer is incorrect. Persons at risk are usually family members or those reluctant to provide care.
(D) This answer is incorrect. This individual has a vested interest in providing care.
40. A 32-year-old female client is being treated for Guillain- Barre syndrome. She complains of gradually
increasing muscle weakness over the past several days. She has noticed an increased difficulty in
ambulating and fell yesterday. When conducting a nursing assessment, which finding would indicate a
need for immediate further evaluation?

A. Complaints of a headache
B. Loss of superficial and deep tendon reflexes
C. Complaints of shortness of breath
D. Facial paralysis
Answer: C
Explanation:
(A) Headaches are not associated with Guillain-Barre syndrome.
(B) Loss of superficial and deep tendon reflexes is expected with this diagnosis.
(C) Complaints of shortness of breath must be further evaluated. Forty percent of all clients have some
detectable respiratory weakness and should be prepared for a possible tracheostomy. Pneumonia is also a
common complication of this syndrome.
(D) Facial paralysis is expected and is not considered abnormal.
41. Plans for the care of a client with an ulcer caused by emotional problems need to take into
consideration that:
A. His priority needs are limited to medical management
B. There is no real psychological basis for his illness
C. The disorder is a threat to his physical well-being
D. He is unable to participate in planning his care
Answer: C
Explanation:
(A) There may be a medical emergency that takes top priority; however, the basis of the problem is
emotional.
(B) The problem is a physical manifestation of an emotional conflict.
(C) The bleeding ulcer can be life threatening.
(D) For lifestyle change to occur, the client must participate in the planning of his care so that he is
committed to changes that will have positive results.
42. A 55-year-old man has recently been diagnosed with hypertension. His physician orders a low sodium
diet for him. When he asks, "What does salt have to do with high blood pressure?'' the nurse's initial
response would be:
A. "The reason is not known why hypertension is associated with a high-salt diet."
B. "Large amounts of salt in your diet can cause you to retain fluid, which increases your blood pressure."

C. "Salt affects your blood vessels and causes your blood pressure to be high."
D. "Salt is needed to maintain blood pressure, but too much causes hypertension."
Answer: B
Explanation:
(A) This response is untrue.
(B) Decreasing salt intake reduces fluid retention and decreases blood pressure.
(C) Salt does not have an effect on the blood vessels themselves, but on fluid retention, which
accompanies salt intake.
(D) This response is untrue.
43. A client is medically cleared for ECT and is tentatively scheduled for six treatments over a 2week
period. Her husband asks, "Isn't that a lot?" The nurse's best response is:
A. "Yes, that does seem like a lot."
B. "You'll have to talk to the doctor about that. The physician knows what's best for the client."
C. "Six to 10 treatments are common. Are you concerned about permanent effects?"
D. "Don't worry. Some clients have lots more than that."
Answer: C
Explanation:
(A) This response indicates that the nurse is unsure of herself and not knowledgeable about ECT. It also
reinforces the husband's fears.
(B) This response is "passing the buck" unnecessarily. The information needed to appropriately answer the
husband's question is well within the nurse's knowledge base.
(C) The most common range for affective disorders is 6-10 treatments. This response confirms and
reinforces the physician's plan for treatment. It also opens communication with the husband to identify
underlying fears and knowledge deficits.
(D) This response offers false reassurance and dismisses the husband's underlying concerns about his wife.
44. A client with IDDM is given IV insulin for a blood glucose level of 520 mg/dL. Life-threatening
complications may occur initially, so the nurse will monitor him closely for serum:
A. Chloride level of 99 mEq/L
B. Sodium level of 136 mEq/L
C. Potassium level of 3.1 mEq/L
D. Potassium level of 6.3 mEq/L

Answer: D
Explanation:
(A) The chloride level is within acceptable limits.
(B) The sodium level is within acceptable limits.
(C) This value indicates hypokalemia, rather than the hyperkalemia that occurs during diabetic
ketoacidosis.
(D) When diabetic ketoacidosis exists, intracellular dehydration occurs and potassium leaves the cells and
enters the vascular system, thus increasing the serum level beyond an acceptable range. When insulin and
fluids are administered, cell walls are repaired and potassium is transported back into the cells. Normal
serum potassium levels range from 3.5-5.0 mEq/L.
45. A 27-year-old primigravida at 32 weeks' gestation has been diagnosed with complete placenta previa.
Conservative management including bed rest is the proper medical management. The goal for fetal
survival is based on fetal lung maturity. The test used to determine fetal lung maturity is:
A. Dinitrophenylhydrazine
B. Metachromatic stain
C. Blood serum phenylalanine test
D. Lecithin-sphingomyelin ratio
Answer: D
Explanation:
(A) Dinitrophenylhydrazine is a laboratory test used to detect phenylketonuria, maple syrup urine disease,
and Lowe's syndrome.
(B) Metachromatic stain is a laboratory test that may be used to diagnose Tay-Sachs and other lipid
diseases of the central nervous system.
(C) The blood serum phenylalanine test is diagnostic of phenylketonuria and can be used for wide-scale
screening.
(D) A lecithin-sphingomyelin ratio of at least 2:1 is indicative of fetal lung maturity, and survival of the
fetus is likely.
46. One week ago, a 21-year-old client with a diagnosis of bipolar disorder was started on lithium 300 mg
po qid. A lithium level is ordered. The client's level is 1.3 mEq/L. The nurse recognizes that this level is
considered to be:
A. Within therapeutic range

B. Below therapeutic range
C. Above therapeutic range
D. At a level of toxic poisoning
Answer: A
Explanation:
(A) This answer is correct. The therapeutic range is 1.0-1.5 mEq/L in the acute phase. Maintenance control
levels are 0.6-1.2 mEq/L.
(B, C) This answer is incorrect. A level of 1.3 mEq/L is within therapeutic range.
(D) This answer is incorrect. Toxic poisoning is usually at the 2.0 level or higher.
47. When discussing the relationship between exercise and insulin requirements, a 26-year-old client with
IDDM should be instructed that:
A. When exercise is increased, insulin needs are increased
B. When exercise is increased, insulin needs are decreased
C. When exercise is increased, there is no change in insulin needs
D. When exercise is decreased, insulin needs are decreased
Answer: B
Explanation:
(A) If the client's insulin is increased when activity level is increased, hypoglycemia may result.
(B) Exercise decreased the blood sugar by promoting uptake of glucose by the muscles. Consequently, less
insulin is needed to metabolize ingested carbohydrates. Extra food may be required for extra activity.
(C) This statement directly contradicts the correct answer and is inaccurate.
(D) When exercise is decreased, the client's insulin dose does not need to be altered unless the blood sugar
becomes unstable.
48. A client had a hemicolectomy performed 2 days ago. Today, when the nurse assesses the incision, a
small part of the abdominal viscera is seen protruding through the incision. This complication of wound
healing is known as:
A. Excoriation
B. Dehiscence
C. Decortication
D. Evisceration
Answer: D

Explanation:
(A) Excoriation is abrasion of the epidermis or of the coating of any organ of the body by trauma,
chemicals, burns, or other causes.
(B) Dehiscence is a partial or complete separation of the wound edges with no protrusion of abdominal
tissue.
(C) Decortication is removal of the surface layer of an organ or structure. It is a type of surgery, such as
removing the fibrinous peel from the visceral pleura in thoracic surgery.
(D) Evisceration occurs when the incision separates and the contents of the cavity spill out.
49. A 3-year-old child is admitted with a diagnosis of possible noncommunicating hydrocephalus. What is
the first symptom that indicates increased intracranial pressure?
A. Bulging fontanelles
B. Seizure
C. Headache
D. Ataxia
Answer: C
Explanation:
(A) Bulging fontanelles are a symptom of increased intracranial pressure in infants.
(B) Seizure is a late sign of increased intracranial pressure.
(C) Headache is a very early symptom of increased intracranial pressure in the child.
(D) Ataxia is a late sign of increased intracranial pressure.
50. Which nursing implication is appropriate for a client undergoing a paracentesis?
A. Have the client void before the procedure.
B. Keep the client NPO.
C. Observe the client for hypertension following the procedure.
D. Place the client on the right side following the procedure.
Answer: A
Explanation:
(A) A full bladder would impede withdrawal of ascitic fluid.
(B) Keeping the client NPO is not necessary.
(C) The client may exhibit signs and symptoms of shock and hypertension.
(D) No position change is needed after the procedure.

51. A client is placed in five-point restraints after exhibiting sudden violence after illegal drug use, and
haloperidol (Haldol) 5 mg IM is administered. After 1 hour, his behavior is more subdued, but he tells the
nurse, "The devil followed me into this room, I see him standing in the corner with a big knife. When you
leave the room, he's going to cut out my heart." The nurse's best response is:
A. "I know you're feeling frightened right now, but I want you to know that I don't see anyone in the
corner."
B. "You'll probably see strange things for a while until the PCP wears off."
C. "Try to sleep. When you wake up, the devil will be gone."
D. "You're probably feeling guilty because you used illegal drugs tonight."
Answer: A
Explanation:
(A) The nurse is the client's link to reality. This response validates the authenticity of the client's
experience by casting doubt on his belief and reinforcing reality.
(B) Although this statement may be literally correct, it is nontherapeutic because it lacks validation.
(C) This response encourages the client to attempt to do something that may be impossible at this time,
offers false reassurance, and reinforces delusional content.
(D) The nurse is making an incorrect assumption about the client's feelings by offering a nontherapeutic
interpretation of the motivation for the client's actions.
52. To facilitate maximum air exchange, the nurse should position the client in:
A. High Fowler
B. Orthopneic
C. Prone
D. Flat-supine
Answer: B
Explanation:
(A) The high Fowler position does increase air exchange, but not to the extent of orthopneic position.
(B) The orthopneic position is a sitting position that allows maximum lung expansion.
(C) The prone position places pressure on diaphragm and does not promote maximum air exchange.
(D) The flat supine position places pressure on diaphragm by abdominal organs and does not promote
maximum air exchange.

53. A 48-year-old client is in the surgical intensive care unit after having had three-vessel coronary artery
bypass surgery yesterday. She is extubated, awake, alert and talking. She is receiving digitalis for atrial
arrhythmias. This morning serum electrolytes were drawn. Which abnormality would require immediate
intervention by the nurse after contacting the physician?
A. Serum osmolality is elevated indicating hemo concentration. The nurse should increase IV fluid rate.
B. Serum sodium is low. The nurse should change IV fluids to normal saline.
C. Blood urea nitrogen is subnormal. The nurse should increase the protein in the client's diet as soon as
possible.
D. Serum potassium is low. The nurse should administer KCl as ordered.
Answer: D
Explanation:
(A) An elevated serum osmolality poses no immediate danger and is not corrected rapidly.
(B) A low serum sodium alone does not warrant changing IV fluids to normal saline. Other assessment
parameters, such as hydration status, must be considered.
(C) A low serum blood urea nitrogen is not necessarily indicative of protein deprivation. It may also be the
result of overhydration.
(D) A low serum potassium potentiates the effects of digitalis, predisposing the client to dangerous
arrhythmias. It must be corrected immediately.
54. A male client is scheduled to have angiography of his left leg. The nurse needs to include which of the
following when preparing the client for this procedure?
A. Validate that he is not allergic to iodine or shellfish.
B. Instruct him to start active range of motion of his left leg immediately following the procedure.
C. Inform him that he will not be able to eat or drink anything for 4 hours after the procedure.
D. Inform him that vital signs will be taken every hour for 4 hours after the procedure.
Answer: A
Explanation:
(A) Angiography, an invasive radiographic examination, involves the injection of a contrast solution
(iodine) through a catheter that has been inserted into an artery.
(B) The client is kept on complete bed rest for 6-12 hours after the procedure. The extremity in which the
catheter was inserted must be immobilized and kept straight during this time.
(C) The contrast dye, iodine, is nephrotoxic. The client must be instructed to drink a large quantity of
fluids to assist the kidneys in excreting this contrast media.

(D) The major complication of this procedure is haemorrhage. Vital signs are assessed every 15 minutes
initially for signs of bleeding.
55. A client had a transurethral resection of the prostate yesterday. He is concerned about the small amount
of blood that is still in his urine. The nurse explains that the blood in his urine:
A. Should not be there on the second day
B. Will stop when the Foley catheter is removed
C. Is normal and he need not be concerned about it
D. Can be removed by irrigating the bladder
Answer: C
Explanation:
(A) Some hematuria is usual for several days after surgery.
(B) The client will continue to have a small amount of hematuria even after the Foley catheter is removed.
(C) Some hematuria is usual for several days after surgery. The client should not be concerned about it
unless it increases.
(D) Irrigating the bladder will not remove the hematuria. Irrigation is done to remove blood clots and
facilitate urinary drainage.
56. The nurse is caring for a client who has had a tracheostomy for 7 years. The client is started on a full
strength tube feeding at 75 mL/hr. Prior to starting the tube feeding, the nurse confirms placement of the
tube in the stomach. The hospital policy states that all tube feeding must be dyed blue. On suctioning, the
nurse notices the sputum to be a blue color. This is indicative of which of the following?
A. The client aspirated tube feeding.
B. The nurse has placed the suction catheter in the esophagus.
C. This is a normal finding.
D. The feeding is infusing into the trachea.
Answer: A
Explanation:
(A) Once the feeding tube placement is confirmed in the stomach, aspiration can occur if the client's
stomach becomes too full. When suctioning the trachea, if secretions resemble tube feeding, the client has
aspirated the feeding.
(B) Because the trachea provides direct access to a client's airway, it would not be possible to place the
catheter in the esophagus.

(C) Blue-colored sputum is never considered a normal finding and should be reported and documented.
(D) The nurse confirmed placement of the feeding tube in the stomach prior to initiating the tube feeding;
therefore, it is highly unlikely that the feeding tube would be located in the trachea.
57. A 3-year-old child has had symptoms of influenza including fever, productive cough, nausea, vomiting,
and sore throat for the past several days. In caring for a young child with symptoms of influenza, the
mother must be cautioned about:
A. Giving aspirin and bismuth subsalicylate (Pepto-Bismol) to treat the symptoms
B. Giving clear liquids too soon
C. Allowing the child to come in contact with other children for 3 days
D. The possibility of pneumonia as a complication
Answer: A
Explanation:
(A) Aspirin should never be given to children with influenza because of the possibility of causing Reye's
syndrome. Pepto- Bismol is also classified as a salicylate and should be avoided.
(B) Depending on the severity of symptoms, the child may be receiving IV therapy or clear liquids.
(C) The disease has a 1-3 day incubation period and affected children are most infectious 24 hours before
and after the onset of symptoms.
(D) Although viral pneumonia can be a complication of influenza, this would not be an initial priority.
58. A male client is considering having laser abdominal surgery and asks the nurse if there is any
advantage in having this type of surgery? The nurse will respond based on the knowledge that laser
surgery:
A. Has a smaller postoperative infection rate than routine surgery
B. Will eliminate the need for preoperative sedation
C. Will result in less operating time
D. Generally eliminates problems with complications
Answer: A
Explanation:
(A) A lower postoperative infection has been documented as a result of laser therapy versus routine
surgery.
(B) Clients will still need preoperative sedation to facilitate anxiety reduction.
(C) Operating time may actually increase in some laser surgeries.

(D) The client must still be observed for postoperative complications.
59. Based on your knowledge of genetic inheritance, which of these statements is true for autosomal
recessive genetic disorders?
A. Heterozygotes are affected.
B. The disorder is always carried on the X chromosome.
C. Only females are affected.
D. Two affected parents always have affected children.
Answer: D
Explanation:
(A) The term heterozygote refers to an individual with one normal and one mutant allele at a given locus
on a pair of homologous chromosomes. An individual who is heterozygous for the abnormal gene does not
manifest obvious symptoms.
(B) Disorders carried on either the X or Y sex chromosome are referred to as sex-linked recessive.
(C) Either sex may be affected by autosomal recessive genetic disorders because the responsible allele can
be on any one of the 46 chromosomes.
(D) If both parents are affected by the disorder and are not just carriers, then all their children would
manifest the same disorder.
60. The paediatric nurse charts that the parents of a 4-yearold child are very anxious. Which observation
would indicate to the nurse unhealthy coping by these parents:
A. Discussing their needs with the nursing staff
B. Discussing their needs with other family members
C. Seeking support from their minister
D. Refusing to participate in the child's care
Answer: D
Explanation:
(A, B, C) These methods are healthy ways of dealing with anxiety.
(D) Participation minimizes feelings of helplessness and powerlessness. It is important that parents have
accurate information and that they seek support from sources available to them.
61. The doctor has ordered a restricted fluid intake for a 2- year-old child with a head injury. Normal fluid
intake for a child of 2 years is:

A. 900 mL/24 hr
B. 1300 mL/24 hr
C. 1600 mL/24 hr
D. 2000 mL/24 hr
Answer: C
Explanation:
(A, B, D) These values are incorrect. Normal intake for a child of 2 years is about 1600 mL in 24 hours.
(C) This value is correct. Normal intake for a child of 2 years is about 1600 mL in 24 hours.
62. A couple is planning the conception of their first child. The wife, whose normal menstrual cycle is 34
days in length, correctly identifies the time that she is most likely to ovulate if she states that ovulation
should occur on day:
A. 14 + 2 days
B. 20 + 2 days
C. 16 + 2 days
D. 22 + 2 days
Answer: B
Explanation:
(A) Ovulation is dependent on average length of menstrual cycle, not standard 14 days.
(B) Ovulation occurs 14 + 2 days before next menses (34 minus 14 does not equal 16).
(C) Ovulation occurs 14 + 2 days before next menses (34 minus 14 equals 20).
(D) Ovulation occurs 14 + 2 days before next menses (34 minus 14 does not equal 22).
63. The nurse is collecting a nutritional history on a 28- year-old female client with iron-deficiency anemia
and learns that the client likes to eat white chalk. When implementing a teaching plan, the nurse should
explain that this practice:
A. Will bind calcium and therefore interfere with its metabolism
B. Will cause more premenstrual cramping
C. Interferes with iron absorption because the iron precipitates as an insoluble substance
D. Causes competition at iron-receptor sites between iron and vitamin B1
Answer: C
Explanation:
(A) Eating chalk is not related to calcium and its absorption.

(B) Poor nutritional habits may result in increased discomfort during premenstrual days, but this is not a
primary reason for the client to stop eating chalk. Premenstrual discomfort has not been mentioned.
(C) Iron is rendered insoluble and is excreted through the gastrointestinal tract.
(D) There is no competition between the two nutrients.
64. A child becomes neutropenic and is placed on protective isolation. The purpose of protective isolation
is to:
A. Protect the child from infection
B. Provide the child with privacy
C. Protect the family from curious visitors
D. Isolate the child from other clients and the nursing staff
Answer: A
Explanation:
(A) The child no longer has normal white blood cells and is extremely susceptible to infection.
(B) There are more appropriate ways to provide privacy, and there is no need to protect the child from
healthy visitors.
(C) Visitors and visiting hours may be at the client's and/or family's request without regard to the isolation
precaution.
(D) The child may have strong positive relationships with other clients or staff. As long as proper
precautions are observed, there is no reason to isolate her from them.
65. A schizophrenic client who is experiencing thoughts of having special powers states that "I am a
messenger from another planet and can rule the earth." The nurse assesses this behavior as:
A. Ideas of reference
B. Delusions of persecution
C. Thought broadcasting
D. Delusions of grandeur
Answer: D
Explanation:
(A) Clients experiencing ideas of reference believe that information from the environment (e.g., the
television) is referring to them.
(B) Clients experiencing delusions of persecution believe that others in the environment are plotting
against them.

(C) Clients experiencing thought broadcasting perceive that others can hear their thoughts.
(D) Clients experiencing delusions of grandeur think that they are omnipotent and have superhuman
powers.
66. Following a gastric resection, which of the following actions would the nurse reinforce with the client
in order to alleviate the distress from dumping syndrome?
A. Eating three large meals a day
B. Drinking small amounts of liquids with meals
C. Taking a long walk after meals
D. Eating a low-carbohydrate diet
Answer: D
Explanation:
(A) Six small meals are recommended.
(B) Liquids after meals increase the time food empties from the stomach.
(C) Lying down after meals is recommended to prevent gravity from producing dumping.
(D) A low-carbohydrate diet will prevent a hypertonic bolus, which causes dumping.
67. A 6-year-old child is attending a paediatric clinic for a routine examination. What should the nurse
assess for while conducting a vision screening?
A. Hearing test
B. Gait
C. Strabismus
D. Papilledema
Answer: C
Explanation:
(A) Hearing should be assessed separately.
(B) Gait should be assessed separately. Client usually remains in one place for vision screening. Gait is
part of neurological assessment.
(C) Strabismus is crossing of eyes or outward deviation, which may cause diplopia or ambylopia. It is
easily assessed during vision screening.
(D) Papilledema is assessed by an ophthalmoscopic examination, which follows vision screening. It is part
of neurological assessment.

68. Which of the following ECG changes would be seen as a positive myocardial stress test response?
A. Hyperacute T wave
B. Prolongation of the PR interval
C. ST-segment depression
D. Pathological Q wave
Answer: C
Explanation:
(A) Hyperacute T waves occur with hyperkalemia.
(B) Prolongation of the P R interval occurs with first degree AV block.
(C) Horizontal ST-segment depression of>1 mm during exercise is definitely a positive criterion on the
exercise ECG test.
(D) Patho-logical Q waves occur with MI.
69. A client with cystic fibrosis exhibits activity intolerance related to the pulmonary problems associated
with his disease. However, he needs to be encouraged to participate in daily physical exercise. The
ultimate aim of exercise is to:
A. Create a sense of well-being and self-worth
B. Help him overcome respiratory infections
C. Establish an effective, habitual breathing pattern
D. Promote normal growth and development
Answer: C
Explanation:
(A) Regular exercise does promote a sense of well-being and self worth, but this is not the ultimate goal of
exercise for this client.
(B) Regular chest physiotherapy, not exercise per se, helps to prevent respiratory infections.
(C) Physical exercise is an important adjunct to chest physiotherapy. It stimulates mucus secretion,
promotes a feeling of well-being, and helps to establish a habitual breathing pattern.
(D) Along with adequate nutrition and minimization of pulmonary complications, exercise does help
promote normal growth and development. However, exercise is promoted primarily to help establish a
habitual breathing pattern.
70. A common complication of cirrhosis of the liver is prolonged bleeding. The nurse should be prepared
to administer?

A. Vitamin C
B. Vitamin K
C. Vitamin E
D. Vitamin A
Answer: B
Explanation:
(A) Vitamin C does not directly affect clotting.
(B) Vitamin K is a fat-soluble vitamin that depends on liver function for absorption. Vitamin K is essential
for clotting.
(C) Vitamin E does not directly affect clotting.
(D) Vitamin A does not directly affect clotting.
71. The nurse in the mental health center is instructing a depressed client about the dietary restrictions
necessary in taking her medication, which is a monoamine oxidase (MAO) inhibitor. Which of the
following is she restricting from the client's diet?
A. Cream cheese
B. Fresh fruits
C. Aged cheese
D. Yeast bread
Answer: C
Explanation:
(A) Cream cheese does not contain tyramine, which might cause a hypertensive crisis.
(B) Fresh fruits do not contain tyramine, which might cause a hypertensive crisis.
(C) Aged or matured cheese combined with a monoamine oxidase predisposes the client to a hypertensive
crisis.
(D) Bread products raised with yeast do not contain tyramine.
72. A 67-year-old client will be undergoing a coronary arteriography in the morning. Client teaching about
post procedure nursing care should include that:
A. Bed rest with bathroom privileges will be ordered
B. He will be kept NPO for 8-12 hours
C. Some oozing of blood at the arterial puncture site is normal
D. The leg used for arterial puncture should be kept straight for 8-12 hours

Answer: D
Explanation:
(A) Bed rest will be ordered for 8-12 hours post procedure. Flexing of the leg at the arterial puncture site
will occur if the client gets out of bed, and this is contraindicated after arteriography.
(B) The client will be able to eat as soon as he is alert enough to swallow safely and that will depend on
what medications are used for sedation during the procedure.
(C) Oozing at the arterial puncture site is not normal and should be closely evaluated.
(D) The leg where the arterial puncture occurred must be kept straight for 8-12 hours to minimize the risk
of bleeding.
73. The nurse is assessing breath sounds in a bronchovesicular client. She should expect that:
A. Inspiration is longer than expiration
B. Breath sounds are high pitched
C. Breath sounds are slightly muffled
D. Inspiration and expiration are equal
Answer: D
Explanation:
(A) Inspiration is normally longer in vesicular areas.
(B) High pitched sounds are normal in bronchial area.
(C) Muffled sounds are considered abnormal.
(D) Inspiration and expiration are equal normally in this area, and sounds are medium pitched.
74. Parents should be taught not to prop the bottle when feeding their infants. In addition to the risk of
choking, it puts the infant at risk for:
A. Otitis media
B. Asthma
C. Conjunctivitis
D. Tonsillitis
Answer: A
Explanation:
(A) Because the eustachian tube is short and straight in the infant, formula that pools in the back of the
throat attacks bacteria which can enter the middle ear and cause an infection.
(B) Asthma is not associated with propping the bottle.

(C) Conjunctivitis is an eye infection and not associated with propping the bottle.
(D) Tonsillitis is usually a result of pharyngitis and not propping the bottle.
75. A client undergoes a transurethral resection, prostate (TURP). He returns from surgery with a threeway continuous Foley irrigation of normal saline in progress. The purpose of this bladder irrigation is to
prevent:
A. Bladder spasms
B. Clot formation
C. Scrotal edema
D. Prostatic infection
Answer: B
Explanation:
(A) The purpose of bladder irrigation is not to prevent bladder spasms, but to drain the bladder and
decrease clot formation and obstruction.
(B) A three-way system of bladder irrigation will cleanse the bladder and prevent formation of blood clots.
A catheter obstructed by clots or other debris will cause prostatic distention and hemorrhage.
(C) Scrotal edema seldom occurs after TURP. Bladder irrigation will not prevent this complication.
(D) Prostatic infection seldom occurs after TURP. Bladder irrigation will not prevent this complication.
76. Priapism may be a sign of:
A. Altered neurological function
B. Imminent death
C. Urinary incontinence
D. Reproductive dysfunction
Answer: A
Explanation:
(A) Priapism in the trauma client is due to the neurological dysfunction seen in spinal cord injury. Priapism
is an abnormal erection of the penis; it may be accompanied by pain and tenderness. This may disappear as
spinal cord edema is relieved.
(B) Priapism is not associated with death.
(C) Urinary retention, rather than incontinence, may occur.
(D) Reproductive dysfunction may be a secondary problem.

77. The healthcare team determines that an elderly client has had progressive changes in memory over the
last 2 years that have interfered with her personal, social, or occupational functioning. Her memory,
learning, attention, and judgment have all been affected in some way. These symptoms describe which of
the following conditions?
A. Dementia
B. Parkinsonism
C. Delirium
D. Mania
Answer: A
Explanation:
(A) These changes are common characteristics of dementia.
(B) Parkinson's disease affects the muscular system. Progressive memory changes are not presenting
symptoms.
(C) Delirium includes an altered level of consciousness, which is not found in dementia.
(D) Mania includes symptoms of hyperactivity, flight of ideas, and delusions of grandeur.
78. After instructing a female client on circumcision care, the nursery nurse asks her to restate some of the
key points covered. Which statement shows that the client will properly care for her son's circumcision?
A. "I'll make sure I soak the gauze with warm water first, before I take it off each time."
B. "I'll make sure that I report any drainage around where they operated."
C. "I'll apply alcohol to the area daily to clean it and prevent any infection."
D. "I'll keep a close watch on it for a day or two."
Answer: A
Explanation:
(A) Before petrolatum gauze is removed, it should be soaked with warm water to prevent trauma to
adherent tissues.
(B) A yellow exudate often forms normally over the surgical site. Only if it becomes foul-smelling and
purulent would it need to be reported.
(C) Alcohol should never be used on the site; this would be extremely painful to the infant.
(D) Special care and observance should continue until the site is completely covered with clean, pink
granulation tissue, which could take 7-10 days.

79. The nurse and prenatal client discuss the effects of cigarette smoking on pregnancy. It would be correct
for the nurse to explain that with cigarette smoking there is increased risk that the baby will have:
A. A low birth weight
B. A birth defect
C. Anemia
D. Nicotine withdrawal
Answer: A
Explanation:
(A) Women who smoke during pregnancy are at increased risk for miscarriage, preterm labor, and IUGR in
the fetus.
(B) Although smoking produces harmful effects on the maternal vascular system and the developing fetus,
it has not been directly linked to fetal anomalies.
(C) Smoking during pregnancy has not been directly linked to anemia in the fetus.
(D) Smoking during pregnancy has not been linked to nicotine withdrawal symptoms in the newborn.
80. Respiratory function is altered in a 16-year-old asthmatic. Which of the following is the cause of this
alteration?
A. Altered surfactant production
B. Paradoxical movements of the chest wall
C. Increased airway resistance
D. Continuous changes in respiratory rate and depth
Answer: C
Explanation:
(A) Altered surfactant production is found in sudden infant death syndrome.
(B) Paradoxical breathing occurs when a negative intrathoracic pressure is transmitted to the abdomen by a
weakened, poorly functioning diaphragm.
(C) Asthma is characterized by spasm and constriction of the airways resulting in increased resistance to
airflow.
(D) If the pulmonary tree is obstructed for any reason, inspired air has difficulty overcoming the resistance
and getting out. The rate of respiration increases in order to compensate, thus increasing air exchange.
81. The FHR pattern in a laboring client begins to show early decelerations. The nurse would best respond
by:

A. Notifying the physician
B. Changing the client to the left lateral position
C. Continuing to monitor the FHR closely
D. Administering O2 at 8 L/min via face mask
Answer: C
Explanation:
(A) Early decelerations are reassuring and do not warrant notification of the physician.
(B) Because early decelerations is a reassuring pattern, it would not be necessary to change the client's
position.
(C) Early decelerations warrant the continuation of close FHR monitoring to distinguish them from more
ominous signs.
(D) O2 is not warranted in this situation, but it is warranted in situations involving variable and/or late
decelerations.
82. A female client decides on haemodialysis. She has an internal vascular access device placed. To ensure
patency of the device, the nurse must:
A. Assess the site for leakage of blood or fluids
B. Auscultate the site for a bruit
C. Assess the site for bruising or hematoma
D. Inspect the site for color, warmth, and sensation
Answer: B
Explanation:
(A) This is an internal device. Assessment of the site should include assessing for swelling, pain, warmth,
and discoloration. This measure does not assess patency.
(B) The presence of a bruit indicates good blood flow through the device.
(C) The nurse should inspect the site for bruising or hematoma; however, this measure does not assure
patency of the device.
(D) The nurse should inspect the vascular access site frequently for signs of infection. However, this does
not assure patency.
83. A client has a chest tube placed in his left pleural space to re-expand his collapsed lung. In a closedchest drainage system, the purpose of the water seal is to:
A. Prevent air from entering the pleural space

B. Prevent fluid from entering the pleural space
C. Provide a means to measure chest drainage
D. Provide an indicator of respiratory effort
Answer: A
Explanation:
(A) A chest tube extends from the pleural space to a collection device. The tube is placed below the surface
of the saline so that air cannot enter the pleural space.
(B) Fluid may enter the pleural space as a result of injury or disease. A chest tube may drain fluid from the
pleural space, but the water seal is not involved in this.
(C) Chest drainage should be measured, but the water seal is not involved in this.
(D) Fluctuations in the tube in the water-sealed bottle will give an indication of respiratory effort, but that
is not the purpose of the water seal.
84. A 42-year-old male client has been treated at an alcoholic rehabilitation center for physiological
alcohol dependence. The nurse will be able to determine that he is preparing for discharge and is
effectively coping with his problem when he shares with her the following information:
A. "I know that I will not ever be able to socially drink alcohol again and will need the support of the AA
group."
B. "I know that I can only drink one or two drinks at social gatherings in the future, but at least I don't have
to continue AA."
C. "I really wasn't addicted to alcohol when I came here, I just needed some help dealing with my
divorce."
D. "It really wasn't my fault that I had to come here. If my wife hadn't left, I wouldn't have needed those
drinks."
Answer: A
Explanation:
(A) The client has insight into the severity of his alcohol addiction and has chosen one of the most
effective treatment strategies to support him-Alcoholics Anonymous.
(B) The client is still using denial and is not dealing with his alcohol addiction.
(C) The client is exhibiting denial about his alcohol addiction and projecting blame on his divorce.
(D) The client is projecting blame onto his wife for being in the hospital while still denying his alcohol
addiction.

85. Which of the following should be included in discharge teaching for a client with hepatitis C?
A. He should take aspirin as needed for muscle and joint pain.
B. He may become a blood donor when his liver enzymes return to normal.
C. He should avoid alcoholic beverages during his recovery period.
D. He should use disposable dishes for eating and drinking.
Answer: C
Explanation:
(A) Aspirin is hepatotoxic, may increase bleeding, and should be avoided.
(B) Blood should not be donated by a client who has had hepatitis C because of the possibility of
transmission of disease.
(C) Alcohol is detoxified in the liver.
(D) Hepatitis C is not spread through the oral route.
86. A physician's order reads: 0.25 normal saline at 50 mL/hr until discontinued. The nurse is using a micro
drip tubing set. How many drops per minute should the nurse administer?
A. 1 gtt/min
B. 5 gtt/min
C. 50 gtt/min
D. 100 gtt/min
Answer: C
Explanation:
(A) This answer is a miscalculation.
(B) This answer is a miscalculation.
(C) 50 gtt/min.
(D) This answer is a miscalculation.
87. A 48-year-old male client is hospitalized with mild ascites, bruising, and jaundice. He has a 20year
history of alcohol abuse. The client is diagnosed with cirrhosis. His serum ammonia level is high,
indicating hepatic encephalopathy. He has esophageal varices. Which of the following may cause the
varices to rupture?
A. Lifting heavy objects
B. Walking briskly
C. Ingestion of barbiturates

D. Ingestion of antacids
Answer: A
Explanation:
(A) Lifting heavy objects will increase intrathoracic pressure, thus placing the client at risk for rupturing
esophageal varices.
(B, C, D) This activity will not cause an increase in intrathoracic pressure.
88. The nurse is admitting an infant with bacterial meningitis and is prepared to manage the following
possible effects of meningitis:
A. Constipation
B. Hypothermia
C. Seizure
D. Sunken fontanelles
Answer: C
Explanation:
(A) Constipation may occur if the child is dehydrated, but it is not directly associated with meningitis.
(B) It is more likely the child will have fever.
(C) Seizure is often the initial sign of meningitis in children and could become frequent.
(D) It is more likely the child will have bulging fontanelles.
89. A male client received a heart-lung transplant 1 month ago at a local transplant center. While visiting
the nursing center to have his blood pressure taken, he complains of recent weakness and fatigue. He also
tells the nurse that he is considering stopping his cyclosporine because it is expensive and is causing his
face to become round. He fears he will catch viruses and be more susceptible to infections. The nurse
responds to this last statement by explaining that cyclosporine:
A. Is given to prevent rejection and makes him less susceptible to infection than other oral corticosteroids
B. Is available at discount pharmacies for a reduced price
C. Is usually not necessary after the first year following transplantation
D. May initially cause weakness, dizziness, and fatigue, but these side effects will gradually resolve
themselves
Answer: A
Explanation:

(A) Cyclosporine is the immunosuppressive drug of choice. It provides immunosuppression but does not
lower the white blood cell count; therefore, the client is less susceptible to infection.
(B) Cyclosporine is available at discount pharmacies. The cost may be absorbed by health insurance, or
Medicare, if the client is eligible. However, this statement does not address the entire problem verbalized
by the client.
(C) Immunosuppressive agents will be taken for the client's entire life because rejection can occur at any
time.
(D) These side effects do not necessarily resolve in time; however, the client may adapt.
90. After the RN is finished the initial assessment of a newborn baby and after the initial bonding between
the newborn and the mother has taken place in the delivery room, the RN will bring the newborn to the
well-baby nursery. Before the newborn is taken from the delivery room and brought to the well-baby
nursery, the RN makes sure that which of the following interventions was completed?
A. The physician verifies the exact time of birth.
B. The nurse counts the instruments and sponges with the scrub nurse.
C. The nurse in stills prophylactic ointment in the conjunctival sacs of the newborn's eyes.
D. The nurse makes sure the mother and her newborn have been tagged with identical bands.
Answer: D
Explanation:
(A) The delivery room personnel are responsible for verifying time of birth.
(B) The scrub and circulating nurses count sponges and instruments.
(C) This intervention is done in the nursery.
(D) Tagging the mother and infant with identical bands is of utmost importance. The mother wears one
band, and the newborn wears two. Identical numbers on the three bands provide identification for the
newborn and the birth mother. Every time the newborn is brought to the mother after delivery, those bands
are checked to be sure that the numbers are identical.
91. A client has been in labor 10 hours and is becoming very tired. She has dilated to 7 cm and is at 0
station with the fetus in a right occipito posterior position. She is complaining of severe backache with
each contraction. One comfort measure the nurse can employ is to:
A. Place her in knee-chest position during the contraction
B. Use effleurage during the contraction
C. Apply strong sacral pressure during the contraction

D. Have her push with each contraction
Answer: C
Explanation:
(A) This measure is inappropriate. The knee-chest position is employed to take pressure off the cord.
(B) Effleurage is a comfort measure but not the one that will contribute most to the relief of backache
caused by a posterior position.
(C) Sacral pressure will counteract the pressure created by the position of the fetal head.
(D) The client is not completely dilated. Pushing is contraindicated until the second stage of labor.
92. A client returned to the unit following a pneumonectomy. As the nurse is assessing her incision, she
notices fresh blood on the dressing. The nurse should first:
A. Reinforce the dressing.
B. Continue to monitor the dressing.
C. Notify the physician.
D. Note the time and amount of blood.
Answer: C
Explanation:
(A) The dressing should not be reinforced without first notifying the physician. The decision may be made
by the physician to reinforce the dressing after assessing the amount of bleeding.
(B) Blood on the dressing is unusual and should make the nurse aware that something more than
continuing to monitor the dressing should be done.
(C) The physician should be notified immediately, because if the bleeding persists, the client may have to
be taken back to surgery.
(D) The time and amount of blood do need to be recorded after the physician is notified.
93. A 48-year-old female client is going to have a cholecystectomy in the morning. In planning for her
postoperative care, the nurse is aware that a priority nursing diagnosis for her will be high risk for:
A. Knowledge deficit
B. Urinary retention
C. Impaired physical mobility
D. Ineffective breathing pattern
Answer: D
Explanation:

(A) The client may have a knowledge deficit, but reducing the risk for knowledge deficit is not a priority
nursing diagnosis postoperatively.
(B) The client will have a Foley catheter for a day or two after surgery. Urinary retention is usually not a
problem once the Foley catheter is removed.
(C) A client having a cholecystectomy should not be physically impaired. In fact, the client is encouraged
to begin ambulating soon after surgery.
(D) Because of the location of the incision, the client having a cholecystectomy is reluctant to breathe
deeply and is at risk for developing pneumonia. These clients have to be reminded and encouraged to take
deep breaths.
94. To appropriately monitor therapy and client progress, the nurse should be aware that increased
myocardial work and O2 demand will occur with which of the following?
A. Positive inotropic therapy
B. Negative chronotropic therapy
C. Increase in balance of myocardial O2 supply and demand
D. Afterload reduction therapy
Answer: A
Explanation:
(A) Inotropic therapy will increase contractility, which will increase myocardial O 2 demand.
(B) Decreased heart rate to the point of bradycardia will increase coronary artery filling time. This should
be used cautiously because tachycardia may be a compensatory mechanism to increase cardiac output.
(C) The goal in the care of the MI client with angina is to maintain a balance between myocardial O2
supply and demand.
(D) Decrease in systemic vascular resistance by drug therapy, such as IV nitro-glycerine or nitroprusside,
or intra-aortic balloon pump therapy, would decrease myocardial work and O2 demand.
95. A gravida 2 para 1 client is hospitalized with severe preeclampsia. While she receives magnesium
sulfate (MgSO4) therapy, the nurse knows it is safe to repeat the dosage if :
A. Deep tendon reflexes are absent
B. Urine output is 20 mL/hr
C. MgSO4 serum levels are >15 mg/dL
D. Respirations are >16 breaths/min
Answer: D

Explanation:
(A) MgSO4 is a central nervous system depressant. Loss of reflexes is often the first sign of developing
toxicity.
(B) Urinary output at 16 breaths/min indicate that toxic levels of magnesium have not been reached.
Medication administration would be safe.
96. The physician orders haloperidol 5 mg IM stat for a client and tells the nurse that the dose can be
repeated in 1-2 hours if needed. The most likely rationale for this order is:
A. The client will settle down more quickly if he thinks the staff is medicating him
B. The medication will sedate the client until the physician arrives
C. Haloperidol is a minor tranquilizer and will not overuse date the client
D. Rapid neuroleptization is the most effective approach to care for the violent or potentially violent client
Answer: D
Explanation:
(A) If the client could think logically, he would not be paranoid. In fact, he is probably suspicious of the
staff, too. Newly admitted clients frequently experience high levels of anxiety, which can contribute to
delusions.
(B) The goal of pharmacological intervention is to calm the client and assist with reality-based thinking,
not to sedate him.
(C) Haloperidol is a neuroleptic and antipsychotic drug, not a minor tranquilizer.
(D) Haloperidol is a high-potency neuroleptic and first-line choice for rapid neuroleptization, with low
potential for sedation.
97. A 27-year-old primigravida stated that she got up from the chair to fix dinner and bright red blood was
running down her legs. She denies any pain previously or currently. The client is very concerned about
whether her baby will be all right. Her vital signs include P 120 bpm, respirations 26 breaths/min, BP
104/58 mm Hg, temperature 98.2_F, and fetal heart rate 146 bpm. Laboratory findings revealed
haemoglobin 9.0 g/dL, haematocrit 26%, and coagulation studies within normal range. On admission, the
peri pad she wore was noted to be half saturated with bright red blood. A medical diagnosis of placenta
previa is made. The priority nursing diagnosis for this client would be:

A. Decreased cardiac output related to excessive bleeding
B. Potential for fluid volume excess related to fluid resuscitation
C. Anxiety related to threat to self
D. Alteration in parenting related to potential fetal injury
Answer: A
Explanation:
(A) Based on the client's history, presence of bright red vaginal bleeding, and haemoglobin value on
admission, the priority nursing diagnosis would be decreased cardiac output related to excessive bleeding.
(B) This nursing diagnosis is a potential problem that does not exist at the present time, and therefore is not
the priority problem.
(C) The client's expressed anxiety is for her child. The fetus will remain physiologically safe if the
decreased cardiac output is resolved.
(D) Initial spontaneous bleeding with placenta previa is rarely life threatening to the mother or the fetus.
Delivery of the fetus will be postponed until fetal maturity is achieved and survival is likely.
98. A client has chronic obstructive pulmonary disease. She is slowly losing weight, and her daughter is
very concerned about increasing her nutrition. The nurse helps the daughter devise a plan of care for her
mother. The plan of care should include which of the following interventions to promote nutrition?
A. Offer her oral hygiene before and after meals.
B. Encourage her to consume milk products.
C. Encourage her to engage in an activity before a meal to stimulate her appetite.
D. Restrict her fluid intake to three glasses of water a day.
Answer: A
Explanation:
(A) Clients with respiratory diseases are generally mouth breathers. Cleaning the oral cavity may improve
the client's appetite, increase her feelings of well-being, and remove the taste and odor of sputum.
(B) Milk causes thick sputum; therefore, milk products would not be beneficial for this client.
(C) Exercise prior to a meal would require increased O2 consumption and most likely would decrease the
client's ability to eat.
(D) Clients with respiratory diseases need increased fluid to liquefy secretions.

99. In admitting a client to the psychiatric unit, the nurse must explain the rules and regulations of the unit.
A client with antisocial personality disorder makes the following remark, "Forget all those rules. I always
get along well with the nurses." Which nursing response to him would be most effective?
A. "OK, don't listen to the rules. See where you end up."
B. "I'm pleased that you get along so well with the staff. You must still know and abide by the rules."
C. "It is irrelevant whether you get along with the nurses."
D. "I'm not the other nurses. You better read the rules yourself."
Answer: B
Explanation:
(A) This answer is incorrect. A nurse should be an appropriate role model. Threats are not appropriate. No
limit setting was stated.
(B) This answer is correct. The nurse made a positive statement followed by a simple, clear, concise setting
of limits.
(C) This answer is incorrect. It appears to have a negative connotation. There was no limit setting.
(D) This answer is incorrect. The nurse obviously responded in a negative manner. Learning takes place
more readily when one is accepted, not rejected. No limits were set.
100. A 14-year-old teenager is demonstrating behavior indicative of an obsessive-compulsive disorder. She
is obsessed with her appearance. She will not leave her room until her hair, clothes, and makeup are
perfect. She always dresses immaculately. Recently, she expressed disgust over her appearance after she
gained 5 lb. After observing a marked weight loss over a 2-week period, her mother suspects that she is
experiencing bulimia. She eats everything on her plate, then runs to the bathroom. In interviewing the
teenager, she discusses in great detail all of the events leading to her bulimia, but not her feelings. What
defense mechanism is she using?
A. Dissociation
B. Intellectualization
C. Rationalization
D. Displacement
Answer: B
Explanation:
(A) Dissociation is separating a group of mental processes from consciousness or identity, such as multiple
personalities. That is not evident in this situation.

(B) Intellectualization is excessive use of reasoning, logic, or words usually without experiencing
associated feelings. This is the defense mechanism that this client is using.
(C) Rationalization is giving a socially acceptable reason for behavior rather than the actual reason. She is
discussing events, not reasons.
(D) Displacement is a shift of emotion associated with an anxiety-producing person, object, or situation to
a less threatening object.
101. A 27-year-old healthy primigravida is brought to the labor and birthing room by her husband at 32
weeks' gestation. She experienced a sudden onset of painless vaginal bleeding. Following an ultrasound
examination, the diagnosis of bleeding secondary to complete placenta previa is made. Expected
assessment findings concerning the abdomen would include:
A. A rigid, board like abdomen
B. Uterine atony
C. A soft relaxed abdomen
D. Hypertonicity of the uterus
Answer: C
Explanation:
(A) A rigid, board like abdomen is an assessment finding indicative of placenta abruptio.
(B) A cause of post birth haemorrhage is uterine atony. With placenta previa, uterine tone is within normal
range.
(C) The placenta is located directly over the cervical os in complete previa. Blood will escape through the
os, resulting in the uterus and abdomen remaining soft and relaxed.
(D) In placenta abruptio, hypertonicity of the uterus is caused by the entrapment of blood between the
placenta and uterine wall, a retroplacental bleed. This does not exist in placenta previa.
102. The nurse assesses a client on the second postpartum day and finds a dark red discharge on the peri
pad. The stain appears to be about 5 inches long. Which of the following correctly describes the character
and amount of lochia?
A. Lochia alba, light
B. Lochia serosa, heavy
C. Lochia granulosa, heavy
D. Lochia rubra, moderate
Answer: D

Explanation:
(A) Lochia alba occurs approximately 10 days after birth and is yellow to white. A discharge is classified
as light when less than a 4-inch stain exists.
(B) Lochia serosa is pink to brown and occurs 34 days after delivery. A stain is classified as heavy when a
peri pad is saturated in 1 hour.
(C) Lochia granulosa is not a proper classification.
(D) Lochia rubra is red, consisting mainly of blood, debris, and bacteria, and lasts from the time of
delivery to 3-4 days afterward. A stain is classified as moderate when less than a 6-inch stain exists.
103. Which of the following blood values would require further nursing action in a newborn who is 4
hours old?
A. Haemoglobin 17.2 g/dL
B. Platelets 250,000/mm3
C. Serum glucose 30 mg/dL
D. White blood cells 18,000/mm3
Answer: C
Explanation:
(A) The normal range for haemoglobin in the newborn is 17-19 g/dL; 17.2 g/dL is within normal limits.
(B) A normal value range for platelets in the newborn is 150,000-400,000 mm3; 250,000/mm3 is within
normal range.
(C) A serum glucose of 30 mg/dL in the first 72 hours of life is indicative of hypoglycemia and warrants
further intervention.
(D) On the day of birth, a white blood cell count of 18 ,000-40,000/mm3 is normal in the newborn.
104. A client was not using his seat belt when involved in a car accident. He fractured ribs 5, 6, and 7 on
the left and developed a left pneumothorax. Assessment findings include :
A. Crackles and paradoxical chest wall movement
B. Decreased breath sounds on the left and chest pain with movement
C. Rhonchi and frothy sputum
D. Wheezing and dry cough
Answer: B
Explanation:

(A) Crackles are caused by air moving through moisture in the small airways and occur with pulmonary
edema. Paradoxical chest wall movement occurs with flail chest when a segment of the thorax moves
outward on inspiration and inward on expiration.
(B) Decreased breath sounds occur when a lung is collapsed or partially collapsed. Chest pain with
movement occurs with rib fractures.
(C) Rhonchi are caused by air moving through large fluid-filled airways. Frothy sputum may occur with
pulmonary edema.
(D) Wheezing is caused by fluid in large airways already narrowed by mucus or bronchospasm. Dry cough
could indicate a cardiac problem.
105. MgSO4 is ordered IV following the established protocol for a client with severe PIH. The anticipated
effects of this therapy are anticonvulsant and:
A. Vasoconstrictive
B. Vasodilative
C. Hypertensive
D. Antiemetic
Answer: B
Explanation:
(A) An anticonvulsant effect is the goal of drug therapy for PIH. However, we would not want to increase
the vasoconstriction that is already present. This would make the symptoms more severe.
(B) An anticon-vulsant effect and vasodilation are the desired outcomes when administering this drug.
(C) An anticonvulsant effect is the goal of drug therapy for PIH; however, hypertensive drugs would
increase the blood pressure even more.
(D) An anticonvulsant effect is the goal of drug therapy for PIH. MgSO4 is not classified as an antiemetic.
Antiemetics are not indicated for PIH treatment.
106. The nurse is caring for a 6-week-old girl with meningitis. To help her develop a sense of trust, the
nurse should:
A. Give her a small soft blanket to hold
B. Give her good perineal care after each diaper change
C. Leave the door open to her room
D. Pick her up when she cries
Answer: D

Explanation:
(A) A soft blanket may be comforting, but it is not directed toward developing a sense of trust.
(B) Good perineal care is important, but it is not directed toward developing a sense of trust.
(C) An infant with meningitis needs frequent attention, but leaving the door open does not foster trust.
(D) Consistently picking her up when she cries will help the child feel trust in her caregivers.
107. The nurse working in a prenatal clinic needs to be alert to the cardinal signs and symptoms of PIH
because:
A. Immediate treatment of mild PIH includes the administration of a variety of medications
B. Psychological counseling is indicated to reduce the emotional stress causing the blood pressure
elevation
C. Self-discipline is required to control caloric intake throughout the pregnancy
D. The client may not recognize the early symptoms of PIH
Answer: D
Explanation:
(A) Mild PIH is not treated with medications.
(B) Emotional stress is not the cause of blood pressure elevation in PIH.
(C) Excessive caloric intake is not the cause of weight gain in PIH.
(D) The client most frequently is not aware of the signs and symptoms in mild PIH.
108. A client at 9 weeks' gestation comes for an initial prenatal visit. On assessment, the nurse discovers
this is her second pregnancy. Her first pregnancy resulted in a spontaneous abortion. She is 28 years old, in
good health, and works full-time as an elementary school teacher. This information alerts the nurse to
which of the following:
A. An increased risk in maternal adaptation to pregnancy
B. The need for anticipatory guidance regarding the pregnancy
C. The need for teaching regarding family planning
D. An increased risk for subsequent abortions
Answer: B
Explanation:
(A, D) There are no data to support this.
(B) Anticipatory guidance and health maintenance is a first line defense in the promotion of healthy
mothers and healthy babies.

(C) There are no data to support this at this time. This will be a concern later.
109. A mother is unsure about the type of toys for her 17-month-old child. Based on knowledge of growth
and development, what toy would the nurse suggest?
A. A pull toy to encourage locomotion
B. A mobile to improve hand-eye coordination
C. A large toy with movable parts to improve pincer grasp
D. Various large colored blocks to teach visual discrimination
Answer: A
Explanation:
(A) Increased locomotive skills make push-pull toys appropriate for the energetic toddler.
(B) Infants progress from reflex activity through simple repetitive behaviors to imitative behavior. Hand
eye coordination forms the foundation of other movements.
(C) At age 8 months, infants begin to have pincer grasp. Toys that help infants develop the pincer grasp are
recommended for this age group.
(D) Various large colored blocks are suggested toys for infants 6-12 months of age to help visual
stimulation.
110. During an examination, the nurse notes that an infant has diaper rash on the convex surfaces of his
buttocks, inner thighs, and scrotum. Which of the following nursing interventions will be most effective in
resolving the condition?
A. Coating the inflamed areas with zinc oxide
B. Using talcum powder on the inflamed areas to promote drying
C. Removing the diaper entirely for extended periods of time
D. Cleaning the inflamed area thoroughly with disposable wet "wipes" at each diaper change
Answer: C
Explanation:
(A) Zinc oxide is not usually applied to inflamed areas because it contributes to sweat retention.
(B) Talcum powder is of questionable benefit and poses a hazard of accidental inhalation.
(C) Removing the diaper and exposing the area to air and light facilitate drying and healing.
(D) Infants may be sensitive to one or more agents in the wet "wipes." It is better to simply clean with a
wet cloth.

111. The nurse is admitting a client with folic acid deficiency anemia. Which of the following questions is
most important for the nurse to ask the client?
A. "Do you take aspirin on a regular basis?"
B. "Do you drink alcohol on a regular basis?"
C. "Do you eat red meat?"
D. "Have your stools been normal?"
Answer: B
Explanation:
(A) Aspirin does not affect folic acid absorption.
(B) Folic acid deficiency is strongly associated with alcohol abuse.
(C) Because folic acid is a coenzyme for single carbon transfer purines, calves liver or other purines are
the meat sources.
(D) Folic acid does not affect stool character.
112. A client is receiving IV morphine 2 days after colorectal surgery. Which of the following observations
indicate that he may be becoming drug dependent?
A. The client requests pain medicine every 4 hours.
B. He is asleep 30 minutes after receiving the IV morphine.
C. He asks for pain medication although his blood pressure and pulse rate are normal.
D. He is euphoric for about an hour after each injection.
Answer: D
Explanation:
(A) Frequent requests for pain medication do not necessarily indicate drug dependence after complex
surgeries such as colorectal surgery.
(B) Sleeping after receiving IV morphine is not an unexpected effect because the pain is relieved.
(C) A person may be in pain even with normal vital signs.
(D) A subtle sign of drug dependency is the tendency for the person to appear more euphoric than relieved
of pain.
113. When preparing insulin for IV administration, the nurse identifies which kind of insulin to use?
A. NPH
B. Human or pork
C. Regular

D. Long acting
Answer: C
Explanation:
(A, B, D) Intermediate-acting and long-acting preparations contain materials that increase length of
absorption time from the subcutaneous tissues but cause the preparation to be cloudy and unsuitable for IV
use. Human insulin must be given SC.
(C) Only regular insulin can be given IV.
114. A 74-year-old female client is 3 days postoperative. She has an indwelling catheter and has been
progressing well. While the nurse is in the room, the client states, "Oh dear, I feel like I have to urinate
again!" Which of the following is the most appropriate initial nursing response?
A. Assure her that this is most likely the result of bladder spasms.
B. Check the collection bag and tubing to verify that the catheter is draining properly.
C. Instruct her to do Kegel exercises to diminish the urge to void.
D. Ask her if she has felt this way before.
Answer: B
Explanation:
(A) Although this may be an appropriate response, the initial response would be to assure the patency of
the catheter.
(B) The most frequent reason for an urge to void with an indwelling catheter is blocked tubing. This
response would be the best initial response.
(C) Kegel exercises while a retention catheter is in place would not help to prevent a voiding urge and
could irritate the urethral sphincter.
(D) Though the nurse would want to ascertain whether the client has felt the same urge to void before, the
initial response should be to assure the patency of the catheter.
115. Following TURP, which of the following instructions would be appropriate to prevent or alleviate
anxiety concerning the client's sexual functioning?
A. "You may resume sexual intercourse in 2 weeks."
B. "Many men experience impotence following TURP."
C. "A transurethral resection does not usually cause impotence."
D. "Check with your doctor about resuming sexual activity."
Answer: C

Explanation:
(A) Sexual activity should be delayed until cleared by the client's physician.
(B) Although many men experience retrograde ejaculation following prostate surgery, potency is seldom
affected.
(C) Although the client may experience retrograde ejaculation, it will not limit his ability to engage in
sexual intercourse.
(D) Although the client should obtain clearance from his physician before resuming sexual activity, this
statement does not give the client any information or reassurance about future sexual activity or potency
that could decrease his anxiety.
116. The most frequent cause of early postpartum haemorrhage is:
A. Hematoma
B. Coagulation disorders
C. Uterine atony
D. Retained placental fragments
Answer: C
Explanation:
(A) Hematomas, which are the result of damage to a vessel wall without laceration of the tissue, are a
cause, though not the most frequent cause.
(B) Coagulation disorders are among the causes of post partal haemorrhage, but they are less common.
(C) The most frequent causes of haemorrhage in the post partal period are related to an interference with
involution of the uterus. Uterine atony is the most frequent cause, occurring in the first 24 hours after
delivery.
(D) Retained placental fragments are also a cause, although these bleeds usually occur 7-14 days after
delivery.
117. A husband asks if he can visit with his wife on her ECT treatment days and what to expect after the
initial treatment. The nurse's best response is:
A. "You'll have to get permission from the physician to visit. Clients are pretty sick after the first
treatment."
B. "Visitors are not allowed. We will telephone you to inform you of her progress."
C. "There's really no need to stay with her. She's going to sleep for several hours after the treatment."

D. "Yes, you may visit. She may experience temporary drowsiness, confusion, or memory loss after each
treatment."
Answer: D
Explanation:
(A) It is within the nurse's realm of practice to grant visiting privileges according to hospital policy. ECT
treatments do not make clients sick.
(B) Visitors are allowed and encouraged, particularly family members.
(C) Clients are usually awake within 1 hour posttreatment. Drowsiness wanes as the anesthetic wears off.
(D) A family member is encouraged to stay with the client after return to the unit. The nurse has used an
opportunity to do family teaching and allay fears by explaining temporary side effects of the treatment.
118. A client at 6 months' gestation complains of tiredness and dizziness. Her haemoglobin level is 10
g/dL, and her haematocrit value is 32%. Her nutritional intake is assessed as sufficient. The most likely
diagnosis is:
A. Iron-deficiency anemia
B. Physiological anemia
C. Fatigue due to stress
D. No problem indicated
Answer: A
Explanation:
(A) This clinical situation is indicative of iron-deficiency anemia because the client has inadequate
nutritional intake. Her blood volume is increasing faster than her red blood cell volume. Anemia is present
in the second trimester when the hemoglobin level is 2 and the presence of phosphatidylglycerol in amniotic fluid indicate fetal lung maturity.
128. Due to his prolonged history of alcohol abuse, an alcoholic client will most likely have deficiencies of
which of the following nutrients?
A. Vitamin C and zinc
B. Folic acid and niacin
C. Vitamin A and biotin
D. Thiamine and pyroxidine
Answer: D
Explanation:
(A) Chronic alcoholism can lead to deficiencies of B complex vitamins including thiamine and pyroxidine.
(B) Chronic alcoholism can lead to deficiencies of vitamins A, D, K, and B complex.
(C) Chronic alcoholism can lead to deficiencies of vitamins A, D, K, and B complex.
(D) Vitamins A, D, K, and B require bile salts to be absorbed from the gastrointestinal tract. A damaged
liver does not form bile salts.
129. A complication for which the nurse should be alert following a liver biopsy is:
A. Hepatic coma
B. Jaundice
C. Ascites
D. Shock
Answer: D
Explanation:
(A) Hepatic coma may occur in liver disease due to the increased NH3levels, not due to liver biopsy.
(B) Jaundice may occur due to increased bilirubin levels, not due to liver biopsy.
(C) Ascites would occur due to portal hypertension, not due to liver biopsy.
(D) Haemorrhage and shock are the most likely complications after liver biopsy because of already
existing bleeding tendencies in the vascular makeup of the liver.
130. Proper positioning for the child who is in Bryant's traction is:
A. Both hips flexed at a 90-degree angle with the knees extended and the buttocks elevated off the bed

B. Both legs extended, and the hips are not flexed
C. The affected leg extended with slight hip flexion
D. Both hips and knees maintained at a 90-degree flexion angle, and the back flat on the bed
Answer: A
Explanation:
(A) The child's weight supplies the countertraction for Bryant's traction; the buttocks are slightly elevated
off the bed, and the hips are flexed at a 90-degree angle. Both legs are suspended by skin traction.
(B) The child in Buck's extension traction maintains the legs extended and parallel to the bed.
(C) The child in Russell traction maintains hip flexion of the affected leg at the prescribed angle with the
leg extended.
(D) The child in "90-90" traction maintains both hips and knees at a 90-degree flexion angle and the back
is flat on the bed.
131. A female baby was born with talipes equinovarus. Her mother has requested that the nurse assigned to
the baby come to her room to discuss the baby's condition. The nurse knows that the paediatrician has
discussed the baby's condition with her mother and that an orthopedist has been consulted but has not yet
seen the baby. What should the nurse do first?
A. Call the orthopedist and request that he come to see the baby now.
B. Question the mother and find out what the paediatrician has told her about the baby's condition.
C. Tell the mother that this is not a serious condition.
D. Tell the mother that this condition has been successfully treated with exercises, casts, and/or braces.
Answer: B
Explanation:
(A) The nurse should call the orthopedist after assessing the mother's knowledge.
(B) The nurse must first assess the knowledge of the parent before attempting any explanation.
(C) The nurse should assess the mother's knowledge of the baby's condition as the first priority.
(D) This answer is correct, but the priority is B.
132. What specific hormone must be present in serum or urine laboratory tests used to diagnose
pregnancy?
A. Human chorionic gonadotropin
B. Estrogen
C. Fetoprotein

D. Sphingomyelin
Answer: A
Explanation:
(A) Human chorionic gonadotropin is the biochemical basis for pregnancy tests. It is produced by the
placenta to help maintain the corpus luteum. Its levels climb rapidly following conception, peaking at
about 8 weeks and then gradually decreasing to low levels after 16 weeks.
(B) Estrogen does steadily rise throughout pregnancy, increasing to 30 times that of pre-pregnancy levels.
Although estrogen levels do change during pregnancy, it is not used as the main hormone of evaluation in
pregnancy tests.
(C) -Fetoprotein is the major protein in the serum of the embryo. It is initially produced by the yolk sac.
(D) Lecithin and sphingomyelin are two phospholipids of which fetal lung surfactant is composed. Levels
are evaluated to determine fetal lung maturity.
133. In assisting preconceptual clients, the nurse should teach that the corpus luteum secretes progesterone,
which thickens the endometrial lining in which of the phases of the menstrual cycle?
A. Menstrual phase
B. Proliferative phase
C. Secretory phase
D. Ischemic phase
Answer: C
Explanation:
(A) Menses occurs during the menstrual phase, during which levels of both estrogen and progesterone are
decreased.
(B) The ovarian hormone responsible for the proliferative phase, during which the uterine endometrium
enlarges, is estrogen.
(C) The ovarian hormone responsible for the secretory phase is progesterone, which is secreted by the
corpus luteum and causes marked swelling in the uterine endometrium.
(D) The corpus luteum begins to degenerate in the ischemic phase, causing a fall in both estrogen and
progesterone.
134. The nurse is caring for a client with pancreatitis. Which of the following IV medications would the
nurse expect the physician to prescribe for control of pain in this client?
A. Morphine sulfate

B. Kerolac tromethamine (Toradol)
C. Promethazine (Phenergan)
D. Meperidine (Demerol)
Answer: D
Explanation:
(A) Morphine sulfate is contraindicated in clients with pancreatitis because it may cause spasms of the
sphincter of Oddi and increase pancreatic pain.
(B) Ketorolac tromethamine is currently not approved by the Food and Drug Administration for IV use.
(C) Promethazine is a medication that has no analgesic properties.
(D) Meperidine is the drug of choice for clients with pancreatitis. It will not cause spasms at the sphincter
of Oddi, which can lead to increased pancreatic pain.
135. The nurse begins morning assessment on a male client and notices that she is unable to palpate either
of his dorsalis pedis pulses in his feet. What is the first nursing action after assessing this finding?
A. Palpate these pulses again in 15 minutes.
B. Use a Doppler to determine presence and strength of these pulses.
C. Document the finding that the pulses are not palpable.
D. Call the physician and notify the physician of this finding.
Answer: B
Explanation:
(A) Palpating these pulses again in 15 minutes may only result in the same findings.
(B) Any time during an assessment that the nurse is unable to palpate pulses, the nurse should then obtain a
Doppler and assess for presence or absence of the pulse and pulse strength, if a pulse is present.
(C) Pulses may be present and assessed through use of a Doppler. Absence of palpable pulses does not
indicate absence of blood flow unless pulses cannot be located with a Doppler.
(D) The nurse would only call the physician after determining that the pulses are absent by both palpation
and Doppler.
136. A 33-year-old client is diagnosed with bipolar disorder, acute phase. This is her first psychiatric
hospitalization, and she is being evaluated for treatment with lithium. Which of the following diagnostic
tests are essential prior to the initiation of lithium therapy with this client?
A. Haematocrit, haemoglobin, and white blood cell (WBC) count
B. Blood urea nitrogen, electrolytes, and creatinine

C. Glucose, glucose tolerance test, and random blood sugar
D. X-rays, electroencephalogram, and electrocardiogram (ECG)
Answer: B
Explanation:
(A) These are general diagnostic blood studies (usually done on admission), but they are not reliable
indicators of lithium therapy clearance.
(B) These are the primary diagnostic tests to determine kidney functioning. Because lithium is excreted
through the kidneys and because it can be very toxic, adequate renal function must be ascertained before
therapy begins.
(C) These are diagnostic blood tests used to determine the presence of endocrine (not renal) dysfunction.
(D) These are other types of diagnostic procedures used to determine musculoskeletal, neural, and cardiac
(rather than renal) functioning.
137. A 60-year-old woman exhibits forgetfulness, emotional lability, confusion, and decreased
concentration. She has been unable to perform activities of daily living without assistance. After a
thorough medical evaluation, a diagnosis of Alzheimer's disease was made. An appropriate nursing
intervention to decrease the anxiety of this client would include:
A. Allowing the client to perform activities of daily living as much as possible unassisted
B. Confronting confabulations
C. Reality testing
D. Providing a highly stimulating environment
Answer: A
Explanation:
(A) This answer is correct. The more the client is able to control her daily routine, the less anxiety she will
experience.
(B) This answer is incorrect. Confrontation tends to increase anxiety.
(C) This answer is incorrect. Reality testing is an assessment tool. It does not decrease anxiety.
(D) This answer is incorrect. A highly stimulating environment increases distractibility and anxiety.
138. On the third postpartum day, a client complains of extremely tender breasts. On palpation, the nurse
notes a very firm, shiny appearance to the breasts and some milk leakage. She is bottle feeding. The nurse
should initially recommend to her to:

A. Take 2 ibuprofen (Motrin) tablets by mouth now because the baby will be returning for feeding in 20
minutes
B. Allow the infant to breast-feed at the next feeding time to empty the breasts
C. Apply ice packs to the breasts and wear a supportive, well-fitting bra
D. Take a warm shower and express milk from both breasts until empty
Answer: C
Explanation:
(A) Judicious use of analgesics is appropriate with breast engorgement; however, mechanical suppression
would be the initial recommendation.
(B) Breast-feeding every 112-3 hours will reduce and/or prevent breast engorgement. Breast-feeding will
promote milk production, which will compound the distention and stasis of the venous circulation of
engorgement in a bottle feeding mother.
(C) Ice packs reduce milk flow while the snug, supportive bra provides mechanical suppression and
decreases pulling on Cooper's ligament. In addition, breast binders or ace bandages may be used for some
women.
(D) Warmth promotes milk production and may stimulate the letdown reflex. These measures would
contribute to the venous congestion of engorgement.
139. While changing the dressing on a client's central line, the nurse notices redness and warmth at the
needle insertion site. Which of the following actions would be appropriate to implement based on this
finding?
A. Discontinue the central line.
B. Begin a peripheral IV.
C. Document in the nurse's notes and notify the physician after redressing the site.
D. Clean the site well and redress.
Answer: C
Explanation:
(A) The nurse may never discontinue a central line without a physician's order.
(B) The nurse may never initiate a peripheral IV without a physician's order except in an emergency
situation.
(C) The nurse should always document findings and alert the physician to the findings as well. The
physician may then initiate a new central line and order the current central line to be discontinued.
(D) Besides cleaning and redressing, the nurse should always document the findings.

140. A pregnant client comes to the office for her first prenatal examination at 10 weeks. She has been
pregnant twice before; the first delivery produced a viable baby girl at 39 weeks 3 years ago; the second
pregnancy produced a viable baby boy at 36 weeks 2 years ago. Both children are living and well. Using
the GTPAL system to record her obstetrical history, the nurse should record:
A. 3-2-00-2
B. 2-2-0-2-2
C. 3-1-1-0-2
D. 2-1-1-0-2
Answer: C
Explanation:
(A) This answer is an incorrect application of the GTPAL method. One prior pregnancy was a preterm
birth at 36 weeks (T =1, P= 1; not T = 2).
(B) This answer is an incorrect application of the GTPAL method. The client is currently pregnant for the
third time (G = 3, not 2), one prior pregnancy was preterm (T= 1, P= 1; not T= 2), and she has had no prior
abortions (A =0).
(C) This answer is the correct application of GTPAL method. The client is currently pregnant for the third
time (G =3), her first pregnancy ended at term (>37 weeks) (T = 1), her second pregnancy ended preterm
20-33 weeks) (P = 1), she has no history of abortion (A=0), and she has two living children (L = 2).
(D) This answer is an incorrect application of the GTPAL method. The client is currently pregnant for the
third time (G =3, not 2).
141. A 60-year-old male client was hospitalized 3 days ago with the diagnosis of acute anterior wall
myocardial infarction. Today he has been complaining of increasing weakness and shortness of breath.
Crackles in both lung bases are audible on auscultation. He is developing:
A. An extension of his myocardial infarction
B. Pneumonia
C. Pulmonary edema
D. Pulmonary emboli
Answer: C
Explanation:
(A) Extensions of his myocardial infarction would be chest pain unrelieved with nitroglycerin, cardiac
enzyme elevations, and electrocardiographic changes.

(B) Persons with pneumonia may complain of weakness and shortness of breath and have crackles in their
lung bases. However, they would also have sputum production and leukocytosis.
(C) Persons who have had myocardial infarctions
(especially anterior wall) are at risk of developing left ventricular heart failure, which is a major cause of
pulmonary edema. Pulmonary edema is manifest by shortness of breath, weakness, and crackles on
auscultation of the lung fields.
(D) Pulmonary emboli may be accompanied by shortness of breath, weakness, and crackles. However, the
pulmonary hypertension that accompanies pulmonary emboli results in signs of increased systemic venous
pressure as well.
142. After a liver biopsy, the best position for the client is:
A. High Fowler
B. Prone
C. Supine
D. Right lateral
Answer: D
Explanation:
(A) This position does not help to prevent bleeding.
(B) This position does not help to prevent bleeding.
(C) This position does not help to prevent bleeding.
(D) The right lateral position would allow pressure on the liver to prevent bleeding.
143. A 14-year-old boy has a head injury with laceration of his scalp over his ear. The nurse should call the
physician to report:
A. Blood pressure increase from 100/80 to 115/85 after lunch
B. Headache that is unresponsive to acetaminophen (Tylenol)
C. Pulse rate ranges between 68 bpm and 76 bpm
D. Temperature rise to 102F rectally
Answer: D
Explanation:
(A) This change in blood pressure may not be significant and does not indicate a widening pulse pressure,
a late sign of increased ICP. It is important to continue to monitor for change in blood pressure.

(B) Acetaminophen may be ineffective in relieving headache after head injury. Stronger analgesics are
contraindicated because they mask neurological signs and may depress the CNS.
(C) Pulse rates between 68 bpm and 76 bpm are within normal limits for a 14-year-old child. It is
important to monitor for a consistent drop in pulse rate, which is a late sign of increasing ICP.
(D) An elevated temperature is abnormal and requires further assessment and medical intervention. The
temperature may be unrelated to the head injury, but CNS infection is serious and difficult to control.
144. A 67-year-old man had a physical examination prior to beginning volunteer work at the hospital. A
routine chest x-ray demonstrated left ventricular hypertrophy. His blood pressure was 180/110 . He is 45 lb
overweight. His diet is high in sodium and fat. He has a strong family history of hypertension. The client is
placed on antihypertensive medication; a low-sodium, low-fat diet; and an exercise regimen. On his next
visit, compliance would best be determined by:
A. A blood pressure reading of 130/70 with a 5-lb weight loss
B. No side effects from antihypertensive medication and an accurate pill count
C. No evidence of increased left ventricular hypertrophy on chest x-ray
D. Serum blood levels of the antihypertensive medication within therapeutic range
Answer: A
Explanation:
(A) A blood pressure within acceptable range best demonstrates compliance, but weight loss cannot be
accomplished without adherence to medication, diet, and exercise.
(B) Absence of side effects does not indicate compliance with medication. Pill counts can be misleading
because the client can alter pill counts prior to visit.
(C) Left ventricular hypertrophy is not an accurate measure of compliance because hypertrophy frequently
does not decrease even with pharmacological management.
(D) Therapeutic blood levels measure the drug level at the time of the test. There is no indication of
compliance several days before testing.
145. The physician has ordered that a daily exercise program be instituted by a client with type I diabetes
following his discharge from the hospital. Discharge instructions about exercise should include which of
the following?
A. Exercise should be performed 30 minutes before meals.
B. A snack may be needed before and/or during exercise.
C. Hyperglycemia may occur 2-4 hours after exercise.

D. The blood glucose level should be 100 mg or below before exercise is begun.
Answer: B
Explanation:
(A) Exercise should not be performed before meals because the blood sugar is usually lower just prior to
eating; therefore, there is an increased risk for hypoglycemia.
(B) Exercise lowers blood sugar levels; therefore, a snack may be needed to maintain the appropriate
glucose level.
(C) Exercise lowers blood sugar levels.
(D) Exercise lowers blood sugar levels. If the blood glucose level is 100 mg or below at the start of
exercise, the potential for hypoglycemia is greater.
146. Stat serum electrolytes ordered for a client in acute renal failure revealed a serum potassium level of
6.4. The physician is immediately notified and orders 50 mL of dextrose and 10 U of regular insulin IV
push. The nurse administering these drugs knows the Rationale for this therapy is to:
A. Remove the potassium from the body by renin exchange
B. Protect the myocardium from the effects of hypokalemia
C. Promote rapid protein catabolism
D. Drive potassium from the serum back into the cells
Answer: D
Explanation:
(A) Sodium polystyrene sulfonate (Kayexalate), a cation exchange resin, exchanges sodium ions for
potassium ions in the large intestine reducing the serum potassium.
(B) Calcium is administered to protect the myocardium from the adverse effects of hyperkalemia. Serum
levels reflect hyperkalemia.
(C) Rapid catabolism releases potassium from the body tissue into the bloodstream. Infection and
hyperthermia increase the process of catabolism.
(D) The administration of dextrose and regular insulin IV forces potassium back into the cells decreasing
the potassium in the serum.
147. A 26-year-old female client presents at 10 weeks' gestation. She currently is a G3 1-0-1-1. Her mother
and grandmother have heart disease. Her grandmother also has insulin-dependent diabetes. The client's
previous delivery was a term female infant weighing 9 lb 13 oz. The client is 5 ft 6 inches tall and her

current weight is 130 lb. Based on her history, she is at risk for developing diabetes in pregnancy. Which of
the following factors places her at risk for gestational diabetes?
A. Age >25 years
B. Maternal weight
C. Previous birth of an infant weighing>9 lb
D. Family history of heart disease
Answer: C
Explanation:
(A) Maternal age older than 30 years is an identified risk factor for diabetes. Age younger than 30 years is
insignificant for diabetes unless there is a familial history of diabetes.
(B) The client's weight is appropriate for her height. Obesity or pregnancy weight >20% of the ideal
weight is a contributing factor to the development of gestational diabetes.
(C) The birth of an infant weighing >9 lb (4000 g) is an identified risk factor for gestational diabetes.
(D) A familial history of heart disease is insignificant in the development of diabetes. However, a familial
history of type II diabetes mellitus is identified as a risk factor in the development of diabetes during
pregnancy.
148. When assessing residual volume in tube feeding, the feeding should be delayed if the amount of
gastric contents (residual) exceeds:
A. 20 mL
B. 25 mL
C. 30 mL
D. 50 mL
Answer: D
Explanation:
(A) A residual volume of 20 mL is not excessive.
(B) A residual volume of 25 mL is not excessive.
(C) A residual volume of 30 mL is not excessive.
(D) Tube feedings should be withheld and physician notified for residual volumes of 50-100 mL.
149. A 5-year-old child was recently diagnosed as having acute lymphoid leukemia. She is hospitalized for
additional tests and to begin a course of chemotherapy designed to induce a remission. She is scheduled to

have a bone marrow aspiration tomorrow. She has had a bone marrow test previously and is apprehensive
about having another. Which of the following interventions will be most effective in relieving her anxiety?
A. Explain what will take place and what she will see, feel, and hear.
B. Remind her that she has had this procedure before and that it is nothing to be afraid of.
C. Tell her not to worry about it, that it will be over soon and she can join her friends in the playroom.
D. Give her a big hug and tell her that she is a big girl now and that she will do just fine.
Answer: A
Explanation:
(A) Even though the child has had the procedure before, she will probably need additional explanations
and emotional support.
(B) The fact that the child has had the procedure before and possibly found it painful or uncomfortable
may increase, not relieve, her stress.
(C) This intervention does nothing to reassure the child and may well prevent her from expressing her
feelings.
(D) This does not prepare the child for the test and burdens her with the expectation that she act bigger and
braver than she is.
150. A female client was employed as a client care technician in a haemodialysis unit. She recently began
to experience extreme fatigue, being able to sleep for 16-20 hours at a time. She also noted that her urine
was tea colored, which she rationalized was a result of the vitamins she began taking to alleviate fatigue.
She was diagnosed with hepatitis B.
After a brief hospital stay, she is discharged to her parent's home. Her mother asks the nurse if any
precautions are necessary to prevent transmission to the client's family. The nurse explains necessary
precautions, which include:
A. Isolation of the client from the remainder of the family
B. Separate bathroom facilities if possible; if not, then cleansing daily of the facilities with a chloride
solution
C. No necessary precautions because she is beyond the contagious phase
D. Laundering clothes separately in cold water with a chloride solution
Answer: B
Explanation:
(A) Isolation is not necessary, even in the acute phase.

(B) Separate bathroom facilities are recommended. If unavailable, daily cleansing with a chloride solution
is recommended.
(C) Precautions continue to be necessary while the client is in the active phase of hepatitis.
(D) Clothes are to be laundered separately in hot water with a chloride solution.
151. A 29-year-old client delivered her fifth child by the Lamaze method and developed a post partal
haemorrhage in the recovery room. What are the initial symptoms of shock that she may experience?
A. Marked elevation in blood pressure, respirations, and pulse
B. Decreased systolic pressure, cold skin, and anuria
C. Rapid pulse; narrowed pulse pressure; cool, moist skin
D. No urinary output, tachycardia, and restlessness
Answer: C
Explanation:
(A) Early shock does not exhibit the symptom of marked elevation in blood pressure. A narrowing of the
pulse pressure is indicative of early shock.
(B) Anuria is a clinical finding in late shock.
(C) All of these clinical findings are congruent with early shock.
(D) Absence of urinary output is a clinical finding in the late phase of shock.
152. A successful executive left her job and became a housewife after her marriage to a plastic surgeon.
She started doing volunteer work for a charity organization. She developed pain in her legs that advanced
to the point of paralysis. Her physicians can find no organic basis for the paralysis. The client's behavior
can be described as:
A. Housework phobia
B. Malingering
C. Conversion reaction
D. Agoraphobia
Answer: C
Explanation:
(A) A typical phobia does not result in physical symptoms (i.e., paralysis).
(B) Malingering is pretending to be ill. This person has a true paralysis.
(C) A conversion reaction is a physical expression of an emotional conflict. It has no organic basis.
(D) Agoraphobia is fear of public places.

153. A pregnant client experiences spontaneous rupture of membranes. The first nursing action is to:
A. Assess the client's respirations
B. Notify the physician
C. Auscultate fetal heart rate
D. Transfer to delivery suite
Answer: C
Explanation:
(A) Immediately following membrane rupture, the fetus is at risk for complications, not necessarily the
mother.
(B) The physician is notified after the nurse completes an assessment of the mother's and fetus's
conditions.
(C) Rupture of membranes facilitates fetal descent. A potential complication is cord prolapse, which is
assessed by auscultating fetal heart rate.
(D) Rupture of membranes does not necessarily indicate readiness to deliver.
154. A client's wife is concerned over his behavior in recent months. He has been diagnosed with
Parkinson's disease, and she is telling his nurse that he has been doing "strange things." The nurse
reassures the wife that the following behavior is normal with Parkinson's disease:
A. "Your husband will experience some periods of muscle flaccidity. Be sure to make him sit down during
these periods."
B. "Your husband may move his hands in motions that look like he is rolling a pill between his fingers."
C. "Twitching of the muscles is to be expected and can occur at any time during the day."
D. "Parkinson's disease causes severe pain in the joints. You should give your husband Tylenol at those
times."
Answer: B
Explanation:
(A) Clients with Parkinson's disease generally experience stiffness and rigid movement.
(B) Pill-rolling movements are a symptom experienced by the Parkinson client.
(C) Twitching of the muscles is not an expected symptom of Parkinson's disease.
(D) Parkinson's disease does not cause joint pain. Mild muscular pain may be present.

155. A 22-year-old single woman was admitted to the psychiatric hospital by her mother, who reported
bizarre behavior. Except for going to work, she spends all her time in her room and expresses concern over
neighbors spying on her. She has fears of the telephone being "bugged." Her diagnosis is schizophrenia.
One nurse per shift is assigned to work with the client. The primary reason for this plan would be to:
A. Protect her from suicide
B. Enable her to develop trust
C. Supervise her medication regimen
D. Involve her in groups for social interaction
Answer: B
Explanation:
(A) Suicide is a greater risk in depression than in schizophrenia.
(B) The client is suspicious and needs help to develop trust, which is basic to her improvement.
(C) Although she will be taking medication, drug therapy would not necessitate consistency in the nurses
assigned.
(D) A suspicious client should have limited exposure to groups, because group participation increases
discomfort.
156. The nurse is interviewing a client with a diagnosis of possible abdominal aortic aneurysm.
Which of the following statements will be reflected in the client's chief complaint?
A. "I've been having a dull pain at the upper left shoulder."
B. "My legs have been numb for three months."
C. "I've only been urinating three times a day lately."
D. "I don't remember anything in particular, I just haven't felt well."
Answer: D
Explanation:
(A, B, C) These complaints are not specific signs and symptoms associated with abdominal aortic
aneurysm. If symptoms are present, the aneurysm is expanding or rupture is imminent.
(D) Many clients may experience no symptoms. The only symptom may be a pulsation noted in the
abdomen in the reclining position.
157. A 26-year-old male client is brought by his wife to the emergency department (ED) unconscious.
Blood is drawn for a stat blood count (CBC), fasting blood sugar level, and electrolytes. An indwelling
urinary catheter is inserted. He has a history of type 1 diabetes (insulin dependent diabetes mellitus

[IDDM]). A diagnosis of ketoacidosis is made. Stat lab values reveal a blood sugar level of 520 mg/dL.
Which of the following should the nurse expect to administer in the ER?
A. D50W by IV push
B. NPH insulin SC
C. Regular insulin by IV infusion
D. Sweetened grape juice by mouth
Answer: C
Explanation:
(A) This action would further increase the client's blood sugar.
(B) NPH insulin is an intermediate acting insulin, with an average of 4-6 hours before onset of action. The
client needs insulin that will act immediately. During a keto acidotic state, the client is dehydrated, so any
insulin administered SC will be poorly absorbed.
(C) Regular insulin is the fastest acting-insulin; when given IV, it will immediately act to decrease blood
sugar. Regular insulin is given to decrease blood glucose levels by promoting metabolism of glucose,
inhibiting lipolysis and formation of ketone bodies.
(D) This action would further increase the client's blood sugar.
158. A 43-year-old client is admitted to the hospital with a diagnosis of peripheral vascular disorder. She
arrives in her room via stretcher and requires assistance to move to her bed. The nurse notes that her left
leg is cold to touch. She complains of having recently experienced muscle spasms in that leg. To determine
if these muscle spasms are indicative of intermittent claudication, the nurse would begin her assessment
with the following question:
A. "Would you describe the intensity, duration, and symptoms associated with your pain?"
B. "Do you experience swelling at the end of the day in the affected and unaffected leg?"
C. "Have you had any lesions of the affected leg that have been difficult to heal?"
D. "Do your muscle spasms occur following rest, walking, or exercising?"
Answer: D
Explanation:
(A) Describing pain is an important aspect of the assessment; however, assessing activity preceding muscle
spasms is equally important.
(B) Edema may occur with peripheral vascular disease, but it is not of particular importance in assessing
intermittent claudication.

(C) Lesions may be present with peripheral vascular disease, but they are not an indication of intermittent
claudication.
(D) With intermittent claudication, muscle spasms occur intermittently, mainly with walking and after
exercising. Rest may relieve muscle spasms.
159. A client's behavior is annoying other clients on the unit. He is meddling with their belongings and
dominating the group. The best approach by the nurse is to:
A. Seclude him in his room.
B. Set limits on his behavior.
C. Have his medication increased.
D. Ignore him and tell the other clients that these behaviors are due to his illness and that they should
understand.
Answer: B
Explanation:
(A) This action by the nurse would be punitive.
(B) Consistent limit setting will help the client to know what is acceptable behavior.
(C) This action is not within the nurse's scope of practice.
(D) This could be dangerous to the client and to others and violates other clients' rights.
160. A behavioral modification program is recommended by the multidisciplinary team working with a 15year-old client with anorexia nervosa. A nursing plan of care based on this modality would include:
A. Role playing the client's eating behaviors
B. Restriction to the unit until she has gained 2 lb
C. Encouraging her to verbalize her feelings concerning food and food intake
D. Provision for a high-calorie, high-protein snack between meals
Answer: B
Explanation:
(A) This answer is incorrect. Role playing is based on learning but is not based on the behavioral
modification model.
(B) This answer is correct. The behavioral modification model is based on negative and positive
reinforcers to change behavior.
(C) This answer is incorrect. Verbal catharsis is not an intervention based on behavioral modification.

(D) This answer is incorrect. Although an acceptable nursing intervention, it is not based on behavioral
modification.
161. An alcoholic client who is completing the inpatient segment of a substance abuse program was placed
on disulfiram (Antabuse) drug therapy. What should the nurse include in the discharge instructions?
A. If disulfiram is taken and alcohol ingested, the client experiences nausea, vomiting and elevated blood
pressure.
B. Disulfiram is most effective when prescribed as late as possible in a recovery program.
C. Disulfiram works on the desensitization principle.
D. The effects of disulfiram can be triggered by alcohol 5 days to 2 weeks after the drug is discontinued.
Answer: D
Explanation:
(A) When alcohol is ingested with disulfiram therapy, the client experiences nausea, vomiting, and a
potentially serious drop in blood pressure.
(B) Disulfiram is most successful when used early in the recovery process while the individual makes
major lifestyle changes necessary for long-term recovery.
(C) Disulfiram works on the classical conditioning principle.
(D) The effects of disulfiram can be felt when alcohol is ingested 1-2 weeks after disulfiram is
discontinued.
162. The postpartum nurse should include which of the following instructions to breast-feeding mothers?
A. Limit feeding times for several days to avoid nipple soreness.
B. Wash the nipples with soap and water before and after each feeding.
C. Daily caloric intake should be increased by 500 cal.
D. Breast milk is totally digestible by the baby because it contains lactose.
Answer: C
Explanation:
(A) Limiting initial feeding times will only delay nipple soreness as well as the establishment of the
letdown reflex, thus encouraging engorgement from clogged ducts and ductules.
(B) Soap should be avoided because it may be excessively drying, predisposing nipples to cracking.
(C) For optimal milk production, an additional 500 kcal over maintenance levels are needed daily.
(D) Lipase, not lactose, emulsifies the fat in breast milk, making it almost totally digestible by infants.

163. At her monthly prenatal visit, a client reports experiencing heartburn. Which nursing measure should
be included in her plan of care to help alleviate it?
A. Restrict fluid intake.
B. Use Alka-Seltzer as necessary.
C. Eat small, frequent bland meals.
D. Lie down after eating.
Answer: C
Explanation:
(A) At least eight glasses of fluid per day are encouraged to help dilute stomach contents, thereby
decreasing irritation.
(B) Alka Seltzer contains aspirin, which is irritating to gastric mucosa, and therefore should be avoided.
(C) Small, frequent bland meals help to decrease gastric pressure and to prevent reflux.
(D) Lying down after meals may cause gastric reflux and prevents optimal gastric emptying.
164. A type I diabetic client is diagnosed with cellulitis in his right lower extremity. The nurse would
expect which of the following to be present in relation to his blood sugar level?
A. A normal blood sugar level
B. A decreased blood sugar level
C. An increased blood sugar level
D. Fluctuating levels with a predawn increase
Answer: C
Explanation:
(A) Blood sugar levels increase when the body responds to stress and illness.
(B) Blood sugar levels increase when the body responds to stress and illness.
(C) Hyperglycemia occurs because glucose is produced as the body responds to the stress and illness of
cellulitis.
(D) Blood sugar levels remain elevated as long as the body responds to stress and illness.
165. An 8-year-old boy has been diagnosed with haemophilia. Which of the following diagnostic blood
studies is characteristically abnormal in this disorder?
A. Partial thromboplastin time
B. Platelet count
C. Complete blood count

D. Bleeding time
Answer: A
Explanation:
(A) Partial thromboplastic time measures activity of thromboplastin, which depends on the intrinsic
clotting factors deficient in children who are haemophiliacs.
(B) Platelet counts are normal in haemophilia.
(C) Haemophilia does not affect the complete blood count.
(D) Bleeding times are normal in haemophiliacs. They measure the time interval for the bleeding from
small superficial wounds to cease.
166. The nurse notes hyperventilation in a client with a thermal injury. She recognizes that this may be a
reaction to which of the following medications if applied in large amounts?
A. Neosporin sulfate
B. Mafenide acetate
C. Silver sulfadiazine
D. Povidone-iodine
Answer: B
Explanation:
(A) The side effects of neomycin sulfate include rash, urticaria, nephrotoxicity, and ototoxicity.
(B) The side effects of mafenide acetate include bone marrow suppression, haemolytic anemia,
eosinophilia, and metabolic acidosis. The hyperventilation is a compensatory response to the metabolic
acidosis.
(C) The side effects of silver sulfadiazine include rash, itching, leukopenia, and decreased renal function.
(D) The primary side effect of povidone- iodine is decreased renal function.
167. Nursing care for the parents of a child with a congenital heart defect would include:
A. Encouraging the parents not to tell the child about the seriousness of the congenital heart defect, so the
child will function as normally as possible
B. Acknowledging the fear and concern surrounding their child's health and assisting the parents through
the grieving process as they mourn the loss of their fantasized healthy child
C. Identifying anger and resentment as destructive emotions that serve no purpose
D. Expressing to the parents after the corrective surgery has been completed successfully that all their grief
feelings will resolve

Answer: B
Explanation:
(A) It is important to discuss with parents the need to treat the child as they would any other children, but
they must be truthful and honest with the child about the heart defect. As the child grows older,
explanations can go into greater depth.
(B) Parents of children with congenital heart defects go through a grieving process over the loss of their
"healthy" child. The nurse needs to recognize these feelings and give the parents a role in the child's care
when they are ready.
(C) Anger and resentment are normal feelings that must be dealt with appropriately.
(D) Parents may go through a second grieving process after the repair of the cardiac defect. During this
grieving period, they mourn the loss of the "defective" child who now may be essentially "normal."
168. Three hours postoperatively, a 27-year-old client complains of right leg pain after knee reduction. The
first action by the nurse will be to:
A. Assess vital signs
B. Elevate the extremity
C. Perform a lower extremity neurovascular check
D. Remind the client that he has a client-controlled analgesic pump, and reinstruct him on its use
Answer: C
Explanation:
(A) Vital signs may be altered if there is acute pain or complications related to bleeding or swelling, but
they should not be assessed before checking the affected extremity.
(B) The extremity will be elevated if ordered by the doctor.
(C) Assessment of the postoperative area is important to determine if bleeding, swelling, or decreased
circulation is occurring.
(D) Reinforcement of teaching on use of the client-controlled analgesic pump is important, but not the first
action.
169. A 28-year-old multigravida has class II heart disease. At her prenatal visit at 34 weeks' gestation, all
of the following observations are made. Which would require intervention?
A. Weight gain of 2 kg in 4 weeks
B. Blood pressure of 128/78
C. Subjective data: shortness of breath after showering

D. Ankle edema reported present in late afternoon and evenings
Answer: C
Explanation:
(A) This is not an excessive weight gain indicative of fluid retention.
(B) The blood pressure is within normal range.
(C) Showering should not cause shortness of breath. This could be a sign of cardiac decompensation.
(D) Dependent ankle edema is normal late in the day among pregnant women. Progressive edema would
be a dangerous development.
170. The nurse is teaching a child's parents how to protect the child from lead poisoning. The nurse knows
that a common source of lead poisoning in children is:
A. Dandelion leaves
B. Pencils
C. Old paint
D. Stuffing from toy animals
Answer: C
Explanation:
(A) Dandelion leaves are not a source of lead.
(B) Pencils are not a source of lead poisoning.
(C) Chewing on objects painted before 1960 is a common source of lead poisoning in children. Gasoline is
another source.
(D) Stuffed animals are not a source of lead.
171. A 79-year-old client with Alzheimer's disease is exhibiting significant memory impairment, cognitive
impairment, extremely impaired judgment in social situations, and agitation when placed in a new
situation or around unfamiliar people. The nurse should include the following strategy in the client's care:
A. Maintain routines and usual structure and adhere to schedules.
B. Encourage the client to attend all structured activities on the unit, whether she wants to or not.
C. Ask the client to go to an activity once. If she gives no response right away, change the question around,
asking the same thing.
D. Give the client two or three choices to decide what she wants to do.
Answer: A
Explanation:

(A) Alzheimer's clients cope poorly with changes in routine because of memory deficits. Schedule changes
cause confusion and frustration, whereas adhering to schedules is helpful and supports orientation.
(B) Insisting that the client go to all unit activities may antagonize her and increase her agitation because
of cognitive impairments. It may be better to allow the client time for calming down or distraction rather
than to insist that she attend every activity.
(C) When repeating a question, allow time first for a response; then use the same words the second time to
avoid further confusion.
(D) The nurse should avoid giving several choices at once. Cognitively impaired clients will become more
frustrated with making decisions.
172. The initial focus when providing nursing care for a child with rheumatic fever during the acute phase
of the illness should be to:
A. Maintain contact with her parents
B. Provide for physical and psychological rest
C. Provide a nutritious diet
D. Maintain her interest in school
Answer: B
Explanation:
(A) This goal is helpful, but rest is essential during the acute phase.
(B) Rest is essential for healing to occur and for pain to be relieved.
(C) This goal is important, but rest is essential.
(D) This goal should be part of the plan of care, but it is not the priority during the acute phase.
173. A 23-year-old borderline client is admitted to an inpatient psychiatric unit following an impulsive act
of self-mutilation. A few hours after admission, she requests special privileges, and when these are not
granted, she stands up and angrily shouts that the people on the unit do not care, and she storms across the
room. The nurse should respond to this behavior by:
A. Placing her in seclusion until the behavior is under control
B. Walking up to the client and touching her on the arm to get her attention
C. Communicating a desire to assist the client to regain control, offering a one-to-one session in a quiet
area
D. Confronting the client, letting her know the consequences for getting angry and disrupting the unit
Answer: C

Explanation:
(A) Threatening a client with punitive action is violating a client's rights and could escalate the client's
anger.
(B) Angry clients need respect for personal space, and physical contact may be perceived as a threatening
gesture escalating anger.
(C) Client lacks sufficient self-control to limit own maladaptive behavior; she may need assistance from
staff.
(D) Confronting an angry client may escalate her anger to further acting out, and consequences are for
acting out anger aggressively, not for getting angry or feeling angry.
174. A female client is concerned that she is in a "high-risk" group for the development of acquired
immunodeficiency syndrome (AIDS). She wants to know about the advisability of donating blood.
Which of the following responses is correct?
A. "Individuals who donate blood are at risk of getting the AIDS virus. You should not donate."
B. "It's OK for you to donate because the blood bank has a test that is 100% effective."
C. "You should not donate since it takes time to develop antibodies to the AIDS virus. If you donate blood
before you develop the antibody, you could pass it on in the blood."
D. "It is not a good idea for you to donate. If you have AIDS, the information is made public and could
destroy your personal life."
Answer: C
Explanation:
(A) The AIDS virus cannot be transmitted to the donor through the blood donation procedure.
(B) The test for the AIDS virus is not absolutely foolproof; therefore, it is not wise for a person with
known risk factors to donate blood.
(C) It takes time for antibodies to the AIDS virus to develop. An infected individual could donate
contaminated blood without it testing positive for the virus.
(D) For reasons of confidentiality, information about individuals infected with AIDS is not made public.
175. A 56-year-old client is admitted to the psychiatric unit in a state of total despair. She feels hopeless
and worthless, has a flat affect and very sad appearance, and is unable to feel pleasure from anything. Her
husband has been assisting her at home with the housework and cooking; however, she has not been eating
much, lies around or sits in a chair most of the day, and is becoming confused and thinks her family does

not want her around anymore. In assessing the client, the nurse determines that her behavior is consistent
with:
A. Transient depression
B. Mild depression
C. Moderate depression
D. Severe depression
Answer: D
Explanation:
(A) Transient depression manifests as sadness or the "blues" as seen with everyday disappointments and is
not necessarily dysfunctional.
(B) Mild depression manifests as symptoms seen with grief response, such as denial, sadness, withdrawal,
somatic symptoms, and frequent or continuous thoughts of the loss.
(C) Moderate depression manifests as feelings of sadness, negativism; low self esteem; rumination about
life's failures; decreased interest in grooming and eating; and possibly sleep disturbances. These symptoms
are consistent with dysthymia.
(D) Severe depression manifests as feelings of total despair, hopelessness, emptiness, inability to feel
pleasure; possibly extreme psychomotor retardation; inattention to hygiene; delusional thinking;
confusion; self-blame; and suicidal thoughts. These symptoms are consistent with major depression.
176. A client had a ruptured abdominal aortic aneurysm that was repaired surgically. Her postoperative
recovery progressed without complications, and she is ready for discharge. Client education in preparation
for discharge began 7 days ago on her admission to the nursing unit.
Evaluation of nursing care related to client education is based on evaluation of expected outcomes.
Which statement made by the client would indicate that she is ready for discharge?
A. "I will not drive but ride in the front seat of the car with a seat belt on for my first doctor's
appointment."
B. "When I bathe tomorrow morning, I will be very careful not to get soap on my incision."
C. "I am allowed to exercise by walking for short periods."
D. "Teach my husband about the diet. He'll be doing all the cooking now."
Answer: C
Explanation:

(A) Postoperatively, clients with major abdominal surgery are instructed to avoid driving, riding in the
front seat, and wearing seat belts because any sudden impact may injure a fresh incision. She should ride
in back seat without a seat belt.
(B) Clients should not sit in the tub and allow the incision to soak in water because this may predispose the
client to infection. A short, cool shower would be preferable. Allowing soap to come in contact with the
incision would not harm it and is frequently used as postoperative wound care at home on discharge from
the hospital.
(C) Activity instructions include: avoid sitting for long periods and get exercise by walking. Lifting more
than 5 lb of weight is also prohibited.
(D) The client must also learn her diet. Her husband cooking is probably a temporary measure unless he
did the cooking prior to her hospitalization. A statement such as this may indicate the need for further
exploration of feelings regarding her illness, dependence, and self-care expectations.
177. In caring at home for a child who just ingested a caustic alkali, the nurse would immediately tell the
mother to:
A. Give vinegar, lemon juice, or orange juice
B. Phone the doctor
C. Take the child to the emergency room
D. Induce vomiting
Answer: A
Explanation:
(A) The immediate action is to neutralize the action of the chemical before further damage takes place.
(B) This action should be done after neutralizing the chemical.
(C) This action should be done after neutralizing the chemical.
(D) Never induce vomiting with a strong alkali or acid. Additional damage will be done when the child
vomits the chemical.
178. A 1000-mL dose of lactated Ringer's solution is to be infused in 8 hours. The drop factor for the tubing
is 10 gtt/mL. How many drops per minute should the nurse administer?
A. 125 gtt/min
B. 48 gtt/min
C. 20 gtt/min
D. 21 gtt/min

Answer: D
Explanation:
(A) This answer is a miscalculation.
(B) This answer is a miscalculation.
(C) This answer has not been rounded off to an even number.
(D) 20.8, or 21 gtt/min.
179. Home-care instructions for the child following a cardiac catheterization should include:
A. Notify the physician if a slight bruise develops around the insertion site.
B. Use sponge bathing until stitches are removed.
C. Give aspirin if the child complains of pain at the insertion site.
D. Keep a clean, dry dressing on the insertion site for 2 days.
Answer: B
Explanation:
(A) A small bruise may develop around the insertion site and is not a reason for alarm.
(B) It is best to keep the child out of the bathtub until the sutures are removed.
(C) Acetaminophen, not aspirin, is the drug of choice if there is pain at the insertion site.
(D) The insertion site should be kept clean and dry and open to air.
180. A 16-year-old female client is admitted to the hospital because she collapsed at home while exercising
with videotaped workout instructions. Her mother reports that she has been obsessed with losing weight
and staying slim since cheerleader try-outs 6 months ago, when she lost out to two of her best friends. The
client is 5'4" and weighs 92 lb, which represents a weight loss of 28 lb over the last 4 months. The most
important initial intervention on admission is to:
A. Obtain an accurate weight
B. Search the client's purse for pills
C. Assess vital signs
D. Assign her to a room with someone her own age
Answer: C
Explanation:
(A) On admission, vital signs are the highest priority. Weight is not a vital sign.
(B) Belongings are routinely searched on admission to a psychiatric unit, but this search is not a high
priority.

(C) Vital signs are a high priority when working with self destructive clients.
(D) Room assignment is of low priority.
181. A couple is experiencing difficulties conceiving a baby. The nurse explains basal body temperature
(BBT) by instructing the female client to take her temperature:
A. Orally in the morning and at bedtime
B. Only one time during the day as long as it is always at the same time of day
C. Rectally at bedtime
D. As soon as she awakens, prior to any activity
Answer: D
Explanation:
(A) Monitoring temperature twice a day predicts the biphasic pattern of ovulation.
(B) Prediction of ovulation relies on consistency in taking temperature.
(C) Nightly rectal temperatures are more accurate in predicting ovulation.
(D) Activity changes the accuracy of basal body temperature and ability to detect the luteinizing hormone
surge.
182. Four days after admission for cirrhosis of the liver, the nurse observes the following when assessing a
male client: increased irritability, asterixis, and changes in his speech pattern. Which of the following
foods would be appropriate for his bedtime snack?
A. Fresh fruit
B. A milkshake
C. Saltine crackers and peanut butter
D. A ham and cheese sandwich
Answer: A
Explanation:
(A) High levels of ammonia, a by-product of protein metabolism, can precipitate metabolic
encephalopathy. These clients need a diet high in carbohydrates and bulk.
(B) Metabolic encephalopathy of the brain associated with liver failure is precipitated by elevated
ammonia levels. Ammonia is a by-product of protein metabolism.
(C, D) Metabolic encephalopathy in liver failure is precipitated by elevated ammonia levels. Ammonia is a
by-product of protein metabolism.

183. A postoperative TURP client returns from the recovery room to the general surgery unit and is in
stable condition. One hour later the nurse assesses him and finds him to be confused and disoriented. She
recognizes that this is most likely caused by:
A. Hypovolemic shock
B. Hypokalaemia
C. Hypernatremia
D. Hyponatremia
Answer: D
Explanation:
(A) Early signs of hypovolemic shock include hypotension, tachycardia, tachypnoea, pallor, and
diaphoresis.
(B) Early signs of potassium depletion include muscular weakness or paralysis, tetany, postural
hypotension, weak pulse, shallow respirations, apathy, weak voice, and electrocardiographic changes.
(C) Early signs of an elevated sodium level include dry oral mucous membranes, marked thirst,
hypertension, tachycardia, oliguria or anuria, anxiety, and agitation.
(D) This answer is correct. Important early clinical findings of a decreased sodium concentration include
confusion and disorientation. Hyponatremia can occur after a TURP because absorption during surgery
through the prostate veins can increase circulating blood volume and decrease sodium concentration.
184. A 6-month-old infant who was diagnosed at 4 weeks of age with a ventricular septal defect, was
admitted today with a diagnosis of failure to thrive. His mother stated that he had not been eating well for
the past month. A cardiac catheterization reveals congestive heart failure. All of the following nursing
diagnoses are appropriate. Which nursing diagnosis should have priority?
A. Altered nutrition: less than body requirements related to inability to take in adequate calories
B. Altered growth and development related to decreased intake of food
C. Activity intolerance related to imbalance between oxygen supply and demand
D. Decreased cardiac output related to ineffective pumping action of the heart
Answer: D
Explanation:
(A) Altered nutrition occurs owing to the fatigue from decreased cardiac output associated with congestive
heart failure.
(B) The decreased intake occurs due to fatigue from the altered cardiac output.
(C) Fatigue occurs due to the decreased cardiac output.

(D) The ineffective action of the myocardium leads to inadequate O2 to the tissues, which produces activity
intolerance, altered nutrition, and altered growth and development.
185. A 35-year-old client has returned to her room following surgery on her right femur. She has an IV of
D5 in one half normal saline infusing at 125 mL/hr and is receiving morphine sulfate 10-15 mg IM q4h prn
for pain. She last voided 51/2 hours ago when she was given her preoperative medication. In monitoring
and promoting return of urinary function after surgery, the nurse would:
A. Provide food and fluids at the client's request
B. Maintain IV, increasing the rate hourly until the client voids
C. Report to the surgeon if the client is unable to void within 8 hours of surgery
D. Hold morphine sulfate injections for pain until the client voids, explaining to her that morphine sulfate
can cause urinary retention
Answer: C
Explanation:
(A) Provision of food and fluid promotes bowel elimination. Nutritional needs postoperatively are
determined by the physician, not the client.
(B) Increasing IV fluids postoperatively will not cause a client to void. Any change in rate of
administration of IV fluids should be determined by the physician.
(C) The postoperative client with normal kidney function who cannot void 8 hours after surgery is
retaining urine. The client may need catheterization or medication. The physician must provide orders for
both as necessary.
(D) Although morphine sulfate can cause urinary retention, withholding pain medication will not ensure
that the client will void. The client with uncontrolled pain will probably not be able to void.
186. The physician decides to prescribe both a short-acting insulin and an intermediate-acting insulin for a
newly diagnosed 8-year-old diabetic client. An example of a short-acting insulin is:
A. Novolin Regular
B. Humulin NPH
C. Lente Beef
D. Protamine zinc insulin
Answer: A
Explanation:
(A) Novolin is a short-acting insulin.

(B, C) NPH and Lente are intermediate-acting insulins.
(D) Protamine zinc insulin is a long-acting insulin preparation.
187. The nurse observes that a client has difficulty chewing and swallowing her food. A nursing response
designed to reduce this problem would include:
A. Ordering a full liquid diet for her
B. Ordering five small meals for her
C. Ordering a mechanical soft diet for her
D. Ordering a pureed diet for her
Answer: C
Explanation:
(A) Full liquids would be difficult to swallow if the muscle control of the swallowing act is affected; this is
a probable reason for her difficulties, given her medical diagnosis of multiple sclerosis.
(B) Five small meals would do little if anything to decrease her swallowing difficulties, other than assure
that she tires less easily.
(C) A mechanical soft diet should be easier to chew and swallow, because foods would be more evenly
consistent.
(D) A pureed diet would cause her to regress more than might be needed; the mechanical soft diet should
be tried first.
188. A 44-year-old client had an emergency cholecystectomy 3 days ago for a ruptured gallbladder. She
complains of severe abdominal pain. Assessment reveals abdominal rigidity and distention, increased
temperature, and tachycardia. Diagnostic testing reveals an elevated WBC count. The nurse suspects that
the client has developed:
A. Gastritis
B. Evisceration
C. Peritonitis
D. Pulmonary embolism
Answer: C
Explanation:
(A) Assessment findings for gastritis would reveal anorexia, nausea and vomiting, epigastric fullness and
tenderness, and discomfort.

(B) Evisceration is the extrusion of abdominal viscera as a result of trauma or sutures failing in a surgical
incision.
(C) Peritonitis, inflammation of the peritoneum, can occur when an abdominal organ, such as the
gallbladder, perforates and leaks blood and fluid into the abdominal cavity. This causes infection and
irritation.
(D) Assessment findings of pulmonary embolism would reveal severe substernal chest pain, tachycardia,
tachypnea, shortness of breath, anxiety or panic, and wheezing and coughing often accompanied by bloodtinged sputum.
189. Hypoxia is the primary problem related to near-drowning victims. The first organ that sustains
irreversible damage after submersion in water is the:
A. Kidney (urinary system)
B. Brain (nervous system)
C. Heart (circulatory system)
D. Lungs (respiratory system)
Answer: B
Explanation:
(A) The kidney can survive after 30 minutes of water submersion.
(B) The cerebral neurons sustain irreversible damage after 4-6 minutes of water submersion.
(C) The heart can survive up to 30 minutes of water submersion.
(D) The lungs can survive up to 30 minutes of water submersion.
190. A 22-year-old client is 16 weeks pregnant. She and her husband are expecting their first baby.
The client tells the nurse that her last normal menstrual period was February 16, with 3 days of spotting on
February 17, 18, and 19. The nurse calculates her expected date of delivery to be:
A. November 23rd
B. December 26th
C. September 14th
D. December 9th
Answer: A
Explanation:
(A) Naele's rule is as follows: add 7 days to the 1st day of the last menstrual period, subtract 3 months, and
then add 1 year.

(B) Naele's rule presumes that the woman has a 28-day menstrual cycle, with conception occurring on the
14th day of the cycle. Slight vaginal spotting may occur in early gestation for unknown reasons but is
insignificant in the calculation of Naele's rule.
(C) Naele's rule presumes that the woman has a 28-day menstrual cycle, with conception occurringon the
14th day of the cycle. Slight vaginal spotting may occur in early gestation for unknown reasons but is
insignificant in the calculation of Naele's rule.
(D) Naele's rule presumes that the woman has a 28-day menstrual cycle, with conception occurring on the
14thday of the cycle. Slight vaginal spotting may occur in early gestation for unknown reasons but is
insignificant in the calculation of Naele's rule.
191. Following a fracture of the left femur, a client develops symptoms of osteomyelitis. During the acute
phase of osteomyelitis, nursing care is directed toward:
A. Moving or turning the client's left leg carefully to minimize pain and discomfort
B. Allowing the client out of bed only in a wheelchair or gurney to minimize weight bearing on the left leg
C. Providing the client with a high-protein, high-fiber diet to promote healing
D. Instituting physical therapy to ensure restoration of optimal functioning of the leg
Answer: A
Explanation:
(A) Any movement of his affected limb will cause discomfort to the child.
(B) No weight bearing will be allowed until healing is well underway to avoid pathological fractures.
(C) The child will be anorexic and may experience vomiting. Diet should be simple and high caloric until
appetite returns and symptoms subside.
(D) Physical therapy is instituted only after infection subsides.
192. After several days, an IDDM client's serum glucose stabilizes, and the registered nurse continues
client teaching in preparation for his discharge. The nurse helps him plan an American Diabetes
Association diet and explains how foods can be substituted on the exchange list. He can substitute 1 oz of
poultry for:
A. One frankfurter
B. One ounce of ham
C. Two slices of bacon
D. One-fourth cup dry cottage cheese
Answer: D

Explanation:
(A) A frankfurter is a high-fat meat on the diabetic exchange list.
(B) Ham is a medium-fat meat on the diabetic exchange list, unless it is a center-cut slice.
(C) One strip of bacon equals one fat exchange
rather than ameat exchange. Dietary substitutions should occur within exchange lists and not between
exchange lists.
(D) Diabetic meat-exchange lists are categorized into lean meat foods, medium-fat meats, and high-fat
meats. Cottage cheese (dry, 2% butterfat), one-fourth cup, can substitute for one lean-meat exchange.
193. The client tells the nurse, "I have pain in my left shoulder." This is considered:
A. Evaluation process
B. Objective information
C. Subjective information
D. Complaining
Answer: C
Explanation:
(A) Evaluation process follows a nursing intervention.
(B) Objective information can be measured.
(C) Subjective information is provided by a person.
(D) Client is reporting a symptom that needs to be assessed.
194. Endotracheal tube cuff pressure should never exceed:
A. 10 mm Hg
B. 20 mm Hg
C. 45 mm Hg
D. 60 mm Hg
Answer: B
Explanation:
(A) Pressure <10 mm Hg places the client at risk for aspiration.
(B) Pressure in the endotracheal tube cuff should never exceed 20 mm Hg, because higher pressure places
the client at risk for tracheal erosion.
(C) A pressure of 45 mm Hg is an extremely high pressure in the endotracheal tube cuff. This places the
client at great risk for tracheal erosion.

(D) A pressure of 60 mm Hg is an extremely high pressure in the endotracheal tube cuff. This places the
client at great risk for tracheal erosion.
195. A 40-year-old client has been admitted to the hospital with severe substernal chest pain radiating
down his left arm. The nurse caring for the client establishes the following priority nursing diagnosisAlteration in comfort, pain related to:
A. Increased excretion of lactic acid due to myocardial hypoxia
B. Increased blood flow through the coronary arteries
C. Decreased stimulation of the sympathetic nervous system
D. Decreased secretion of catecholamines secondary to anxiety
Answer: A
Explanation:
(A) Anaerobic metabolism results because the decreased blood supply to the myocardium causes a release
of lactic acid. Lactic acid is an irritant to the myocardial neural receptors, producing chest pain.
(B) Chest pain is caused by a decrease in the O2 supply to the myocardial cells. Treatment modalities for
chest pain are aimed toward increasing the blood flow through coronary arteries.
(C) Chest pain causes an increase in the stimulation of the sympathetic nervous system. This stimulation
increases the heart rate and blood pressure, causing an increase in myocardial workload aggravating the
chest pain.
(D) Chest pain and anxiety cause increased secretion of catecholamines by stimulating the sympathetic
nervous system. This stimulation increases chest pain by increasing the workload of the heart.
196. The nurse working with a client who is out of control should follow a model of intervention that
includes which of the following?
A. Approach the client on a continuum of least restrictive care.
B. Challenge client's behavior immediately with steps to prevent injury to self or others.
C. Leave the aggressive client to himself or herself, and take other clients away.
D. To ensure safety of other clients, place client in seclusion immediately when he or she begins shouting.
Answer: A
Explanation:
(A) Approaching a client's aggressive behavior on a continuum of least restrictive care is in agreement
with his or her rights (i.e., verbal methods to help maintain control, medication, seclusion, and restraints,
as necessary).

(B) Approaching a client in a challenging manner is threatening and inappropriate. A nonchallenging and
calm approach reflects staff in control and may increase client's internal control.
(C) It is inappropriate to leave an aggressive client who is acting out alone. The nurse should acquire
qualified help to prevent client from harm or injury to self or others.
(D) Moving a client to seclusion immediately for shouting is inappropriate. The nurse should offer the
client an opportunity to control self with limit setting. The client should understand that the staff will assist
with control if necessary (i.e., quietly accompany out of environment to decrease stimulation and allow for
verbalization) employing the least restrictive care model of intervention.
197. A 2-year-old toddler is hospitalized with epiglottitis. In assessing the toddler, the nurse would expect
to find:
A. A productive cough
B. Expiratory stridor
C. Drooling
D. Crackles in the lower lobes
Answer: C
Explanation:
(A) A productive cough is not associated with epiglottitis.
(B) Children with epiglottitis seldom have expiratory stridor. Inspiratory stridor is more common due to
edema of the supraglottic tissues.
(C) Because of difficulty with swallowing, drooling often accompanies epiglottitis.
(D) Crackles are not heard in the lower lobes with epiglottitis because the infection is usually confined to
the supraglottic structures.
198. A 24-year-old graduate student recognizes that he has a phobia. He suffers severe anxiety when he is
in darkness. It has altered his lifestyle because he is unable to go to a movie theater, concert, and other
events that may require absence of light. The client is seeking assistance because he is no longer able to
socialize with friends due to his phobia. The psychologist working with him is using desensitization. He
has asked the nursing staff to assist the client in muscle relaxation techniques. What result would indicate
client education has been successful?
A. He enters a movie theater, sits in his chair, and replaces anxiety with relaxation as the theater darkens.
B. He enters a concert, but as the lights dim, he does not experience anxiety.
C. He states that he no longer fears dark places.

D. He takes a part-time job as a photographic assistant. His job necessitates his working in a darkroom.
Answer: A
Explanation:
(A) This situation provides specific evidence that the client is able to integrate muscle relaxation technique
into his lifestyle to alleviate anxiety.
(B) The client may not experience anxiety at the concert, but there is no evidence regarding the technique
that he used to alleviate anxiety.
(C) The client may state he no longer experiences anxiety, but there is no evidence demonstrating this. He
may be denying anxiety to discontinue therapy prematurely.
(D) Does he experience anxiety in the darkroom? He may have taken this job to force himself to deal with
the phobia directly.
199. When planning care for the passive-aggressive client, the nurse includes the following goal:
A. Allow the client to use humor, because this may be the only way this client can express self.
B. Allow the client to express anger by using "I" messages, such as "I was angry when . . .," etc.
C. Allow the client to have time away from therapeutic responsibilities.
D. Allow the client to give excuses if he forgets to give staff information.
Answer: B
Explanation:
(A) Ceasing to use humor and sarcasm is a more appropriate goal, because this client uses these behaviors
covertly to express aggression instead of being open with anger.
(B) Use of "I" messages demonstrates proper use of assertive behavior to express anger instead of passiveaggressive behavior.
(C) Client is expected to complete share of work in therapeutic community because he has often obstructed
other's efforts by failing to do his share.
(D) Client has used conveniently forgetting or withholding information as a passive-aggressive behavior,
which is not acceptable.
200. A 26-year-old client is in a treatment center for aprazolam (Xanax) abuse and continues to manifest
moderate levels of anxiety 3 weeks into the rehabilitation program, often requesting medication for "his
nerves." Included in the client's plan of care is to identify alternate methods of coping with stress and
anxiety other than use of medication. After intervening with assistance in stress reduction techniques,
identifying feelings and past coping, the nurse evaluates the outcome as being met if:

A. Client promises that he will not abuse aprazolam after discharge
B. Client demonstrates use of exercise or physical activity to handle nervous energy following conflicts of
everyday life
C. Client is able to verbalize effects of substance abuse on the body
D. Client has remained substance free during hospitalization and is discharged
Answer: B
Explanation:
(A) This client response does not address stress reduction techniques. Verbal response focuses only on the
problem.
(B) Exercise or physical activity is a common strategy or coping technique used to reduce stress and
anxiety.
(C) Verbalizing effects of substance abuse on the body may help with insight and break through denial, but
it is not a strategy to reduce anxiety.
(D) Remaining substance free does indicate motivation to change lifestyle of substance abuse or
dependence, and it is not a stress reduction strategy in itself.

Document Details

Related Documents

person
Harper Mitchell View profile
Close

Send listing report

highlight_off

You already reported this listing

The report is private and won't be shared with the owner

rotate_right
Close
rotate_right
Close

Send Message

image
Close

My favorites

image
Close

Application Form

image
Notifications visibility rotate_right Clear all Close close
image
image
arrow_left
arrow_right